Вы находитесь на странице: 1из 130

Module 1: SEQUENCES

Lesson 1: Arithmetic Sequences


Learning Competency 1: Generates pattern

I- OBJECTIVES
a. Generate and describe patterns.
b. Identify the next few terms of a sequence.
c. Appreciate sequences as vital part of one’s life.

II - SUBJECT MATTER
Topic: Sequences
Materials: Pictures, Cut-outs of Check and Cross marks, pen and paper
References: Grade 10 Mathematics Teachers’ Guide, pp. 14 – 16
Grade 10 Mathematics Learners’ Guide, pp. 9 – 11
Mathematics III: Concepts, Structures and Methods for High
School by Oronce, Orlando, et.al., pp. 491 – 508

III - PROCEDURE
A. Preliminaries
Motivational Activity:

With Pattern or Without Pattern? That is the Question


.
Identify if each picture below shows a pattern or not. If there is a pattern, put a
check mark (✓) and identify it, otherwise put a cross mark (x).

1. Banderitas
________________________
___
2. Coffee beans

________________________
___

3. Tahong
shells ________________________
___

4. Stack of Stones
In the Beach
________________________
___

1
5. Dates in the
calendar for the
month of July

________________________
___

B. Lesson Proper

1. Teaching/ Modeling

Developmental Activity:
Complete the pattern generated from the previous activity.
a. Color of the Banderitas
Green, Blue, Red, Orange, Yellow, Green, Blue, Red, Orange,
Yellow, _____
b. Number of Coffee beans
4, 6, 8, _____
c. Tahong shells
Close, Open, Close, Open, Open, Close, Open, Open, Open, _____
d. Number of Stack of Stones
1, 2, 3, 4, 5, _____
e. Dates in the calendar for the month of July
3, 10, 17, 24, _____

Illustrative Example 1:
Using the previous example in the number of coffee beans, 4, 6, 8,
10.
This sequence has 4 terms.
The first term, denoted as a1, is 4 which means if the domain n = 1,
then, a1 = 4.
The second term, denoted as a2, is 6 which means if the domain
n = 2, then, a2 = 6.
Then the third term, denoted as a3, is 8 which means if the domain
n = 3, then, a3 = 8.
And the last term or the fourth term, denoted as a4, is 10 which
means if the domain n = 4, then, a4 = 10.

The table below further explains the above example:


domain (n) 1 2 3 4
a1 a2 a3 a4
an
4 6 8 10

2
Illustrative Example 2:
Consider the rule form of the sequence an = 7n – 4.
If we are asked to get the first five terms of the sequence, we have
a1= 7(1) – 4 = 3
a2= 7(2) – 4 = 10
a3= 7(3) – 4 = 17
a4= 7(4) – 4 = 24
a5= 7(5) – 4 = 31

Therefore, the sequence can also be denoted as 3, 10, 17, 24, 31.

2. Analysis

Using the given picture in the previous activity, which of them shows a
pattern?
What pattern is shown in every item?
If the pattern continues in each of the item, what will be the next term?
How do you generate a sequence from a given rule?

3. Guided Practice

Given the following nth term, supply each blank by a correct answer
following the task at the right to generate the pattern.

a. Given an = 2(n+1), list the first 5 terms of the sequence.


Solution:
if n = 1
a__ = 2(__+1) Substitute n
a1 = 2(__) Add the terms inside the parenthesis
a1 = ___ Multiply the factors

Do the same procedure if n = 2, n = 3, n = 4 and n = 5. Then, list the


sequence below.

___, ___, ___, ___, ___

1 𝑛−1
b. Given an = (2) , generate a sequence with 4 terms.
Solution:
if n = 1
1 ___−1
a__ = (2) Substitute the value of n

3
1 ___
a1 = (2) Subtract the exponent

a1 = ____ Simplify the exponent and the fraction

Do the same procedure if n = 2, n = 3 and n = 4 then, list the


sequence below.

___, ___, ___, ___

4. Independent Practice

Complete the table below by substituting the given values of n to a n and


list down the terms of the sequence.

n
an Sequence
1 2 3 4
1.) 3n – 5 -2 1 4 ___ _______________
2.) 3*2n – 1 3 6 ___ 24 _______________
5 5
3.) − 𝑛 -5 ___ − ___ _______________
3
4.) 5 − 𝑛2 ___ 1 -4 ___ _______________
5.) (2𝑛 − 10)2 ___ ___ ___ ___ _______________

5. Generalization

A sequence is a function whose domain is a finite set of positive


integers {1, 2, 3, …, n} or an infinite set {1, 2, 3, …}

Each element or object in the sequence is called term.

A sequence having last term is called finite sequence while a


sequence with no last term is called infinite sequence.

Sequences may come in rule form. These are sequences stated in


general or nth terms.

6. Application

Answer the following problems.


1. Emilia helps her mother in selling “Kalamay Buna” (a delicacy from
Indang). From the money that her mother is giving her, she plans to
save Php25 every week for seven weeks. Form a sequence that will

4
show the amount of money she is saving from the first to seventh
week.
2. The increase in the population of Cavite Province follows a pattern.
That is, 1.5% of its previous year’s population is added to the
present to obtain the next. If the current population of Cavite is
3,000,000, list the province’s population for the next 2 years.
3. The table below shows the cost of renting the Cavite Hall at Island
Cove Resort and Leisure Park in Kawit, Cavite depending on the
number of attendees.
Number of Persons Rental Cost in Peso
20 6200
25 6500
30 6800
35 7100
Jose booked the hall for a birthday party for 40 persons. How much
will he pay?
7. Assessment

Study the following patterns then supply the missing term to complete
the sequence.
1. Jan, Mar, May, Jul, Aug, ___, ____
2. 5, 8, 11, 14, ___, 20, ___...
1 3 5 7
3. 2 , 1, 2 , 2, 2 , ___, 2 , 4, ___, …
4. 3, -6, ___, -24, 48, ___ …
5. 1, 4, 9, 16, ___, 36, 49, 64, ___

IV - ASSIGNMENT
1. Follow-up
a. Give two (2) examples of finite sequence and another two (2)
examples of infinite sequence.
b. The area of the largest square in the figure below is 64 sq. units.
The succeeding square is half the area of its preceding square.
Find the area of the smallest square in the figure.

5
2. Study : Arithmetic Sequence
a. Define (1) arithmetic sequence; and (2) common difference.
b. Study the following sequences then group them according to the
way the pattern is generated.
1, 4, 7, 10, … 9, 12, 15, 18, …
2, -10, 50, -250, … 5, 10, 20, 40, …
2, 6, 10, 14, … 3, 12, 48, 192, …
7, 12, 17, 22, … 4, 11, 18, 25, …
1, 3, 9, 27, … 1, 4, 16, 64, …
c. Observe the things around you. Take a picture of objects forming
a sequence.
d. Identify pictures that show arithmetic sequence.

Electronic Sources: https://en.m.wikipedia.org/wiki/Sequence


www.mathisfun.com/algebra/sequences-series.html

6
Answer Key:

Motivational Activity: With Pattern or Without Pattern? That is the Question.


Identify if each picture below shows a pattern or not. If there is a pattern, put a
check mark (✓) and identify it, otherwise put a cross mark (x).
1. ✓ Green, Blue, Red, Orange, Yellow, Green, Blue, Red, Orange, Yellow
2. ✓ 4, 6, 8
3. ✓ Close, Open, Close, Open, Open, Close, Open, Open, Open,
4. ✓ 1, 2, 3, 4, 5
5. ✓ 3, 10, 17, 24, 31

Developmental Activity: Complete the pattern generated from the previous activity.
1. Green
2. 10
3. Close
4. 6
5. 31

Guided Practice
Given the following nth term, supply each blank by a correct answer following the
task at the right to generate the pattern.
a. Given an = 2(n+1), list the first 5 terms of the sequence.
if n = 1 if n = 2 if n = 3 if n = 4 if n = 5
a1 = 2(1+1) a2 = 2(2+1) a3 = 2(3+1) a4 = 2(4+1) a5 = 2(5+1)
a1 = 2(2) a2 = 2(3) a3 = 2(4) a4 = 2(5) a5 = 2(6)
a1 = 4 a2 = 6 a3 = 8 a4 = 10 a5 = 12

The sequence is 4, 6, 8, 10, 12


1 𝑛−1
b. Given an = (2) , generate a sequence with 4 terms.
if n = 1 if n = 3
1 1−1 1 3−1
a1 = (2) a3 = (2)
1 0 1 2
a1 = ( ) a3 = ( )
2 2
1
a1 = 1 a3 = 4

if n = 2 if n = 4
1 2−1 1 4−1
a2 = (2) a4 = (2)
1 1 1 3
a2 = (2) a4 = (2)
1 1
a2 = 2 a4 = 8

1 1 1
The sequence is 1, , , .
2 4 8
7
Independent Practice
Complete the table below by substituting the given values of n to a n and list down the
terms of the sequence.

1. 7; -2, 1, 4, 7
2. 12; 3, 6,12, 24
5 5 5 5 5
3. −2;−4; −5, − 2 , − 3 , − 4
4. 4; -11; 4, 1, -4, -11
5. 64; 36; 16; 4; 64, 36, 16, 4

Application
Answer the following problems.
1. 25, 50, 75, 100, 125, 150, 175
2. 3045000, 3090675
3. Php 7400

Assessment
Study the following patterns then supply the missing term to complete the sequence.
1. Oct, Dec
2. 17, 23
9
3. 3, 2
4. 12, -96
5. 25, 81

8
Module 1: SEQUENCES
Lesson 1: Arithmetic Sequences
Learning Competency 2: Illustrates an arithmetic sequence

I- OBJECTIVES
a. describe an arithmetic sequence.
b. find the nth term of an arithmetic sequence.
c. appreciate arithmetic sequence in solving real life problems.

II - SUBJECT MATTER
Topic: Arithmetic Sequence
Sub-topic: Finding the nth term of an Arithmetic Sequence
Materials: Picture, Chalk-board, pen and paper
References: Grade 10 Mathematics Teachers’ Guide, pp. 16 – 18
Grade 10 Mathematics Learners’ Guide, pp. 12 – 14
Mathematics III: Concepts, Structures and Methods for High
School by Oronce, Orlando, et.al., pp. 509 – 511

III - PROCEDURE
A. Preliminaries
Motivation:
Saulog Transit Inc. is one of the
many bus transportation companies in the
Philippines servicing routes between
Cavite and Metro Manila, Olongapo or
Baguio City.
One day, on its way back to its
terminal at Mendez, via Aguinaldo
Highway, one (1) passenger went down at
SM City Bacoor, then, another four (4)
passengers went down to Robinson’s
Place Imus, seven (7) passengers went
down to Robinson’s Place Pala-pala and ten (10) passengers went down to
Lourdes Church at Tagaytay.

List down the number of passengers who went down in each places.
Does it form sequence? If it does, how is the sequence formed?

B. Lesson Proper

1. Teaching/ Modeling

Illustrative Example 1:
Using the generated sequence from the previous scenario:

1, 4, 7, 10 Arithmetic Sequence
Subtracting two consecutive
9
terms (i.e.: d = a2 – a1)
4–1 7–4 10 – 7
=3 =3 =3 Common difference (d)

The sequence generated from the given scenario which is 1, 4, 7,


10 is an example of an Arithmetic Sequence because it is formed by
adding a constant number which is 3 to the preceding term to obtain
the next. The constant number 3 is the common difference, denoted
as d, which can be obtained by subtracting two consecutive terms
(d = an – an-1).

Illustrative Example 2:
Find the 10th term of the sequence 3, 8, 13, 18, …
Solution:
Note that a1 = 3, d = 5 and n = 10. Using the formula for the general
term of an arithmetic sequence, we have
𝑎𝑛 = 𝑎1 + (𝑛 − 1)𝑑
𝑎10 = 3 + (10 − 1)5
𝑎10 = 3 + (9)5
𝑎10 = 3 + 45
𝒂𝟏𝟎 = 𝟒𝟖

That is, 3, 8, 13, 18, 23, 28, 33, 38, 43, 48 10 th term

Illustrative Example 3:
In the arithmetic sequence 5, 9, 13, 17, … which term is 401?
Solution:
The problem asks for n when an = 401. From the given sequence, a1
= 5, d = 4 and an = 401. Substituting these values in the formula, we
have
𝑎𝑛 = 𝑎1 + (𝑛 − 1)𝑑
401 = 5 + (𝑛 − 1)4
Solving for n, we have
401 = 5 + 4𝑛 − 4
401 = 4𝑛 + 1
401 − 1 = 4𝑛 + 1 − 1
400 = 4𝑛
1 1
400 ( ) = 4𝑛 ( )
4 4
𝟏𝟎𝟎 = 𝒏

Therefore, 401 is the 100th term.

2. Analysis

How is Arithmetic Sequence formed?


10
How to get the common difference in an arithmetic sequence?
How to find the nth term an arithmetic sequence?

3. Guided Practice

Supply each blank by a correct answer following the task at the right to
solve the question.

a. Find a45 of the sequence 4, 7, 10, 13, 16, …


Given: a1 = ____ ; d = ____ ; n = ____
Solution:
𝑎𝑛 = 𝑎1 + (𝑛 − 1)𝑑
𝑎𝑛 = ___ + (___ − 1)___ substitute a1, n and d
𝑎𝑛 = 4 + (____)3 subtract the terms inside the parenthesis
𝑎𝑛 = 4 + (____) multiply
𝑎𝑛 = _____ add

b. Which term of the arithmetic sequence 7, 14, 21, 28, … is 105?


Given: a1 = ____ ; d = ____ ; an = ____
Solution:
𝑎𝑛 = 𝑎1 + (𝑛 − 1)𝑑
____ = _____ + (𝑛 − 1)____ substitute the given
105 = 7 + ____ − ____ distribute d
105 = ______ subtract the constants in the
right side then apply APE
_____ = 𝑛 apply MPE

4. Independent Practice

Find an for each of the following arithmetic sequence.


1. a1 = 5; d = 4 ; n = 11
2. a1 = 14; d = –3 ; n = 25
3. a1 = 12; d = ½; n = 16
4. –10, –6, –2, 2, 6, … n = 27
5 3
5. 3, 2 , 2, 2 , 1, … n = 28

5. Generalization

An arithmetic sequence is a sequence where every term after the first


is obtained by adding a constant.

11
Common difference (d) is the constant number added to the preceding
term of the arithmetic sequence. It can be calculated by subtracting any
two consecutive term in the arithmetic sequence.

The formula for the general term of an arithmetic sequence is


𝒂𝒏 = 𝒂𝟏 + (𝒏 − 𝟏)𝒅

6. Application

Answer the following problems.


1. You went to a hiking with your friends at Pico de Loro at
Maragondon, Cavite. Upon reaching the summit, you drop a coin.
The coin falls a distance of 4ft for the first seconds, 16ft for the next,
28ft on the third, and so on. Find the distance the coin will fall in 6
seconds?
2. Antonio is studying Chabacano, a native dialect from Cavite City
and Ternate. He started practicing one (1) word for an hour and
decided to add two more words every succeeding hour. If the
pattern continues, how many Chabacano word did he learn in one
day?
3. Rico bought an e-bike at Php29,000. If it depreciates Php500 in
value each year, what will be its value at the end of 10years?
4. Tinapa (smoked fish) is best paired with Atchara (pickled papaya).
Diana, a tinapa vendor in Salinas, Rosario, Cavite, decided to sell
atchara at her store. On the first week, she started to sell 15 atchara
bottles and due to high demand, she decided to add 7 more bottles
on each succeeding weeks. Supposed that the pattern continues,
how may week is needed to sell 57 atchara bottles?
5. A Zumba Program calls for 15 minutes dancing each day for a
week. Each week thereafter, the amount of time spent dancing
increases by 5 minutes per day. In how many weeks will a person
be dancing 60 minutes each day?

7. Assessment

Solve the following questions.


6. Given the sequence 3, 1, –1, –3, …, find a12.
7. Find the 9th term of the arithmetic sequence 12, 24, 36, …
8. If a1 = –17 and d = 4, find a22 of the arithmetic sequence.
9. Find the 16th term of the arithmetic sequence whose first term is 6
and the common difference is 0.25.
10. Which term is 27 in the arithmetic sequence 54, 51, 48, …?

12
IV - ASSIGNMENT
1. Follow-up
a. Can the common difference be negative? If so, describe the
sequence.
b. From the previous assignment, identify which of the following is
an arithmetic sequence then find each common difference.
1, 4, 7, 10, … 9, 12, 15, 18, …
2, -10, 50, -250, … 5, 10, 20, 40, …
2, 6, 10, 14, … 3, 12, 48, 192, …
7, 12, 17, 22, … 4, 11, 18, 25, …
1, 3, 9, 27, … 1, 4, 16, 64, …

2. Study: Arithmetic Mean


a. Definition of Arithmetic Mean.
b. How to insert terms in an arithmetic sequence?

Electronic Sources: http://newsinfo.inquirer.net/567965/name-play-with-maragondon-


peaks
https://www.pinterest.com/kebidodoy/philippine-cuisine-and-
culture/

13
Answer Key:

Motivational Activity:
List down the number of passengers who went down in each places.
1, 4, 7, 10

Does it form sequence?


YES

If it does, how is the sequence formed?


The sequence is formed by adding a constant number which is 3 to the
preceding term.

Guided Practice
Supply each blank by a correct answer following the task at the right to solve
the question.
a. Find a45 of the sequence 4, 7, 10, 13, 16, …
Given: a1 = 4 ; d = 3 ; n = 45
Solution:
𝑎𝑛 = 𝑎1 + (𝑛 − 1)𝑑
𝑎𝑛 = 4 + (45 − 1)3 substitute a1, n and d
𝑎𝑛 = 4 + (44)3 subtract the terms inside the parenthesis
𝑎𝑛 = 4 + (132) multiply
𝑎𝑛 = 136 add

b. Which term of the arithmetic sequence 7, 14, 21, 28, … is 105?


Given: a1 = 7 ; d = 7 ; an = 105
Solution:
𝑎𝑛 = 𝑎1 + (𝑛 − 1)𝑑
105 = 7 + (𝑛 − 1) 7 substitute the given
105 = 7 + 7n − 7 distribute d
105 = 7n subtract the constants in the right side
then apply APE
15 = 𝑛 apply MPE

Independent Practice
Find an for each of the following arithmetic sequence.
1. 44
2. –58
39
3. 2 𝑜𝑟 19.5
4. 94
21
5. − 2 𝑜𝑟 − 10.5

14
Application
Answer the following problems.
1. 64ft
2. 47 Chabacano words
3. Php 24500
4. 7th week or 7weeks
5. 10th week or 10 weeks

Assessment
Solve the following questions.
1. –19
2. 108
3. 67
4. 9.75
5. 10

15
Module 1: SEQUENCES
Lesson 1: Arithmetic Sequences
Learning Competency 3: Determines arithmetic means and nth term of an arithmetic
sequence

I- OBJECTIVES
a. find the missing terms of an arithmetic sequence.
b. Insert a certain number of terms between two given terms of an arithmetic
sequence.
c. appreciate arithmetic means in solving real life problems.

II - SUBJECT MATTER
Topic: Arithmetic Sequence
Sub-topic: Arithmetic Means
Materials: Chalk-board, pen and paper
References: Grade 10 Mathematics Teachers’ Guide, pp. 17
Grade 10 Mathematics Learners’ Guide, pp. 14 – 15
Mathematics III: Concepts, Structures and Methods for High
School by Oronce, Orlando, et.al., pp. 512 – 516
Mathematics III An Integrated Approach by Coronel C. Antonio,
et.al., pp. 63 – 65

III - PROCEDURE
A. Preliminaries
Motivational Activity:

Group the class into two (2) groups then let them find the missing terms in
each of the following arithmetic sequence. The group with highest points after
the game will be the winner.

1. 2, 6, 10, ___, ___, ___


2. 9, 17, ___, ___, ___, 49
3. 7, 9, ___, ___, 15, ___
4. 4, ___, 20, 28, ___, ___
5. 5, ___, ___, 20, 25, ___
6. ___, ___, ___, 3, –1, –5
7. ___, ___, 14, 20, ___, 32
8. ___, 45, 40, ___, ___, 25
9. 4, –4, ___, ___, ___, –36
10. –12, ___, ___, ___, 8, 13

16
B. Lesson Proper

1. Teaching/ Modeling

Developmental Activity:
Follow the instructions below then find a partner to share your answer.
You may use a clean sheet of paper and a pen while doing the activity.

1. Choose two (2) different numbers.


2. Denote the smaller number as x and the larger number as y.
3. Find the mean of this two numbers. That is, add these two number
(𝑥+𝑦)
then divide the sum by 2. In symbols, 2 .
4. Denote the first mean as m2.
5. Now, find the mean of the smaller number x and m2. In symbols,
(𝑥 + 𝑚2 )
.
2
6. Denote the second mean as m1.
7. Then, find the mean of the larger number y and m2. In symbols,
(𝑦+𝑚2 )
.
2
8. Denote the third mean as m3.
9. Lastly, arrange all the numbers in the form x, m1, m2, m3, y.
10. Share your answer with your partner.

Does the result forms arithmetic sequence?


What is its common difference?
What do you call m1, m2, m3?

Illustrative Example 1:
Insert three arithmetic means between 3 and 11.

Solution 1:
We look three numbers m1, m2 and m3 such that 3, m1, m2, m3, 11 is
an arithmetic sequence. In this case, we have a1 = 3, n = 5, a5 = 11.
Using the general formula for arithmetic sequence,
𝑎𝑛 = 𝑎1 + (𝑛 − 1)𝑑
11 = 3 + (5 − 1)𝑑 solve for d
11 = 3 + 4𝑑
11 − 3 = 3 − 3 + 4𝑑
8 = 4𝑑
1 1
8 ( ) = 4𝑑 ( )
4 4
𝒅=𝟐
Since d = 2, so we have
𝑚1 = 𝑎1 + 𝑑 𝑚2 = 𝑚1 + 𝑑 𝑚3 = 𝑚2 + 𝑑
𝑚1 = 3 + 2 = 𝟓 𝑚2 = 5 + 2 = 𝟕 𝑚3 = 7 + 2 = 𝟗
17
Therefore, the three arithmetic means between 3 and 11 are 5, 7, and
9.

Solution 2:
Still, we look three numbers m1, m2 and m3 such that 3, m1, m2, m3, 11
is an arithmetic sequence.
In this case, we need to solve for m2, the mean of a1 = 3 and a5 = 11.
That is,
(𝑎1 + 𝑎5 ) (3 + 11) 14
𝑚2 = = = =𝟕
2 2 2

Now, solve for m1, the mean of a1 = 3 and m2 = 7. That is,


(𝑎1 + 𝑚2 ) (3 + 7) 10
𝑚1 = = = =𝟓
2 2 2
Then, solve for m3, the mean of a5 = 3 and m2 = 7. That is,
(𝑎5 + 𝑚2 ) (11 + 7) 18
𝑚3 = = = =𝟗
2 2 2

Forming the sequence 3, m1, m2, m3, 11, we have 3, 5, 7, 9, 11.

Illustrative Example 2:
The 4th term of an arithmetic sequence is 28 and the 15th term is 105.
Find the common difference and the first term of the sequence.

Solution:
We know that a4 = 28 and a15 = 105. Thus we have
𝑎4 = 𝑎1 + (4 − 1)𝑑
𝑎15 = 𝑎1 + (15 − 1)𝑑

Substituting the given values in the equation, we have


28 = 𝑎1 + (4 − 1)𝑑 (eq. 1)
105 = 𝑎1 + (15 − 1)𝑑 (eq. 2)

Eliminating a1, we subtract (eq. 1) from (eq. 2)


77 = 11d
Therefore, d = 7.

Solving for a1, we substitute d = 7 to (eq. 1)


28 = 𝑎1 + (4 − 1)7
28 = 𝑎1 + (3)7
28 = 𝑎1 + 21
𝒂𝟏 = 𝟕

Therefore, the common difference and the first term of the given
sequence are d = 7 and a1 = 7, respectively.
18
2. Analysis

How did you obtain the missing term of the arithmetic sequence?
Is the common difference necessary to obtain the missing term of the
sequence?
How did you obtain the common difference?
If we cannot solve the common difference by subtracting two
consecutive terms, is there any other way to solve for it?
What is arithmetic mean?
3. Guided Practice

Supply each blank by a correct answer following the task at the right to
answer the question.

a. Insert two terms in the arithmetic sequence 15, ___, ___, 36.
Given: a1 = ____ ; n = ____ ; a4 = ____
Solution:
𝑎𝑛 = 𝑎1 + (𝑛 − 1)𝑑
___ = ___ + (___ − 1)𝑑 substitute a1, n and a4.
(
36 = 15 + ____ 𝑑 ) subtract the terms inside the parenthesis
___ = 3𝑑 apply APE
𝑑 = _____ apply MPE

After solving d, find the second (m1) and the third (m2) term.
𝒎𝟏 = 𝑎1 + 𝑑 = ____ + ____ = _____ substitute a1 and d then add.
𝒎𝟐 = 𝑚1 + 𝑑 = ____ + ____ = _____ substitute m1 and d then add.

b. Insert three arithmetic means between 12 and 56.


Given: a1 = ____; a5 = ____
Solution:
(𝑎 +𝑎 ) (___+___) ___
𝑚2 = 1 2 5 = 2 = 2 = ____ substitute a1 and a5 then solve
for m2.
(𝑎1 +𝑚2) (___+___) ___
𝑚1 = = 2 = 2 = ____ substitute a1 and m2 then solve
2
for m1.
(𝑎5 +𝑚2 ) (___+___) ___
𝑚3 = = 2 = 2 = ____ substitute a5 and m2 then solve
2
for m3.

4. Independent Practice

Answer the following.


1. Insert two arithmetic means between 20 and 38.
19
2. Insert three arithmetic means between 52 and 40.
3. Find the missing terms of the arithmetic sequence 5, ___, ___, ___,
___, 25.
4. Find the missing terms of the arithmetic sequence 0, ___, ___, ___,
___, ___, 15.
5. The fifteenth term of an arithmetic sequence is –3 and the first term
is 25. Find the common difference and the tenth term.

5. Generalization

Arithmetic Means are the terms between any two nonconsecutive


terms of an arithmetic sequence.

It is necessary to solve the common difference of an arithmetic


sequence to insert terms between two nonconsecutive terms of an
arithmetic sequence. The formula for the general term of an arithmetic
sequence, 𝒂𝒏 = 𝒂𝟏 + (𝒏 − 𝟏)𝒅 and the mid-point between two
(𝒙+𝒚)
numbers, 𝟐 can also be used.

6. Application

Answer the following problems.


1. Flower farms in Tagaytay grew different variety of flowers including
anthurium. Monica, a flower arranger, went to Tagaytay to buy
anthurium. She plans to arrange the flowers following an arithmetic
sequence with four (4) layers. If she put one (1) anthurium on the
first layer and seven (7) on the fourth layer, how many anthurium
should be placed on the second and third layer of the flower
arrangement?
2. St. Mary Magdalene Parish Church in Kawit, one of the oldest
churches in Cavite, established in 1624 by Jesuit Missionaries. The
church is made of red bricks preserved for more than a hundred
years. Suppose that the lowest part of the church wall contains five
(5) layers of red bricks, 4bricks on the top and 16bricks on the
bottom layer. Assuming an arithmetic sequence, how many bricks
are there in the 2nd, 3rd and 4th layer of the wall?
3. In some of the Kiddie parties nowadays, Tower Cupcakes were
quite popular because it is appealing and less expensive. In Juan
Miguel’s 1st birthday party, his mother ordered a six (6) layer tower
cupcakes. If the 1st and 4th layer of the tower contains 6 and 21
cupcakes, respectively, how many cupcakes are there in the 6th
layer (bottom) of the tower assuming arithmetic sequence in the
number of cupcakes?

20
7. Assessment

Use the following numbers inside the box to complete the arithmetic
sequence below. You may use a number more than once.

1. 2, ___, ___, 14 6 7 10 11
2. 4, ___, ___, ___, 10
3. 6, ___, ___, ___, 16 12 13 15 18
4. 9, ___, ___, ___, ___, 24
19 21 11/2 17/2
5. ___, 17, ___, ___, 11
27/2 29/2

IV - ASSIGNMENT
1. Follow-up
a. Find the arithmetic mean of –23 and 7.
b. How many numbers are divisible by 9 between 5 and 1000?

2. Study: Sum of Arithmetic Sequence


a. How to find the sum of terms in an arithmetic sequence?
b. Find the sum of the following arithmetic sequence
1, 4, 7, 10, 13, 16, 19, 22, 25
4, 11, 18, 25, 32, 39, 46, 53, 60
2, 6, 10, 14, 18, 22, 26, 30, 34
7, 12, 17, 22, 27, 32, 37, 42, 47
9, 12, 15, 18, 21, 24, 27, 30, 33

Electronic Sources: http://study.com/academy/lesson/arithmetic-mean-definition-


formula-example.html
http://www.mathgoodies.com/lessons/vol8/mean.html
http://www.123rf.com/photo_37149016_group-of-red-anthurium-
flower-in-pot-blooming-in-botanic-farm-anthurium-andraeanum-
araceae-or-arum.html
https://www.pinterest.com/annakarinsund/examens-fest/
http://www.bluedreamer27.com/saint-mary-magdalene-exhibit-in-
kawit-cavite/

21
Answer Key:

Motivational Activity
Group the class into two (2) groups then let them find the missing terms in each of
the following arithmetic sequence. The group with highest points after the game will be the
winner.

1. 14, 18, 22 6. 15, 11, 7


2. 25, 33, 41 7. 2, 8, 26
3. 11, 13, 17 8. 50, 35, 30
4. 12, 36, 44 9. -12, -20, -28
5. 10, 15, 30 10. -7, -2, 3

Guided Practice
Supply each blank by a correct answer following the task at the right to answer the
question.

a. Insert two terms in the arithmetic sequence 15, ___, ___, 36.
Given: a1 = 15 ; n = 4 ; a4 = 36

Solution:
𝑎𝑛 = 𝑎1 + (𝑛 − 1)𝑑
36 = 15 + (4 − 1)𝑑 substitute a1, n and a4.
36 = 15 + (3)𝑑 subtract the terms inside the parenthesis
21 = 3𝑑 apply APE
𝑑=7 apply MPE

After solving d, find the second (m1) and the third (m2) term.
𝒎𝟏 = 𝑎1 + 𝑑 = 15 + 7 = 𝟐𝟐 substitute a1 and d then add.
𝒎𝟐 = 𝑚1 + 𝑑 = 22 + 7 = 𝟐𝟗 substitute m1 and d then add.

b. Insert three arithmetic means between 12 and 56.


Given: a1 = 12; a5 = 56
Solution:
(𝑎 +𝑎 ) (12+56) 68
𝑚2 = 1 2 5 = 2 = 2 = 𝟑𝟒 substitute a1 and a5 then solve for m2.
(𝑎1 +𝑚2) (12+34) 46
𝑚1 = = = = 𝟐𝟑 substitute a1 and m2 then solve for m1.
2 2 2
(𝑎5 +𝑚2 ) (56+34) 90
𝑚3 = = = = 𝟒𝟓 substitute a5 and m2 then solve for m3.
2 2 2

22
Independent Practice
Solve the following.
1. 26, 32
2. 49, 46, 43
3. 9, 13, 17, 21
5 15 25 1 1 1
4. 2.5, 5, 7.5, 10, 12.5 or , 5, , 10, or 2 2 , 5, 7 2 , 10, 12 2
2 2 2
5. d = –2; a10 = –21

Application
Answer the following problems.
1. a2 = 3 flowers , a3 = 5 flowers
2. a2 = 7 bricks, a3 = 10 bricks, a4 = 13 bricks
3. 31 cupcakes

Assessment
Use the following numbers inside the box to complete the arithmetic sequence below.
You may use a number more than once.
1. 6, 10
2. 11/2, 7, 17/2
3. 17/2, 11, 27/2
4. 12, 15, 18, 21
5. 19, 15, 13

23
Module 1: SEQUENCES
Lesson 1: Arithmetic Sequences
Learning Competency 4: Finds the sum of the terms of a given arithmetic sequence

I- OBJECTIVES
a. find the sum of terms of a given arithmetic sequence.
b. solve problems involving the sum of arithmetic sequence.
c. appreciate the sum of arithmetic sequence in solving real life problems.

II - SUBJECT MATTER
Topic: Arithmetic Sequence
Sub-topic: Sum of Arithmetic Sequence
Materials: 25 pieces of empty cans or coins or blocks, chalk-board, pen and paper
References: Grade 10 Mathematics Teachers’ Guide, pp. 19
Grade 10 Mathematics Learners’ Guide, pp. 20 – 21
Mathematics III: Concepts, Structures and Methods for High
School by Oronce, Orlando, et.al., pp. 517 – 522
Mathematics III: An Integrated Approach by Coronel, Antonio C.
et.al., pp. 517 – 522

III - PROCEDURE
A. Preliminaries
Motivational Activity:
Perform the instructions below then answer the questions followed.
1. Form a pyramid of cans with 6 cans in the
first row.
2. Place one (1) fewer cans in each
successive row thereafter.
3. After forming the pyramid, how many
rows does the pyramid have?
4. How many cans are there in each rows?
Does the number of cans in each row
form an arithmetic sequence?
5. How many total cans are there in the pyramid?

B. Lesson Proper

1. Teaching/ Modeling

Illustrative Example 1:
Find the sum of the first 20 terms of the arithmetic sequence 15, 19,
23, 27, …
Solution 1:
We first find a20 by substituting a1 = 15, d = 4 and n = 20 in the
formula 𝑎𝑛 = 𝑎1 + (𝑛 − 1)𝑑
𝑎20 = 15 + (20 − 1)4
𝑎20 = 15 + (19)4
24
𝑎20 = 15 + 76
𝑎20 = 91
Solving for S20, we substitute n = 20, a1 = 15 and an = 91 in the
formula
𝑛
𝑆𝑛 = (𝑎1 + 𝑎𝑛 )
2
20
𝑆20 = (15 + 91)
2
20
𝑆20 = (106)
2
𝑆20 = 10(106)
𝑺𝟐𝟎 = 𝟏𝟎𝟔𝟎

Therefore, the sum of the first 20 terms of the arithmetic sequence


15, 19, 23, 27, … is 1060.

Solution 2:
Substituting a1 = 15, d = 4 and n = 20 in the formula
𝑛
𝑆𝑛 = 2 [2𝑎1 + (𝑛 − 1)𝑑 ], we have
20
𝑆20 = [2(15) + (20 − 1)4]
2
20
𝑆20 = [2(15) + (19)4]
2
20
𝑆20 = [2(15) + 76]
2
20
𝑆20 = (30 + 76)
2
20
𝑆20 = (106)
2
𝑆20 = 10(106)
𝑺𝟐𝟎 = 𝟏𝟎𝟔𝟎

Using an alternative solution, the sum of the first 20 terms of the


arithmetic sequence 15, 19, 23, 27, … is still 1060.

Illustrative Example 2:
How many terms is needed for –3, 2, 7, … to have a sum of 116?
Solution:
Using the formula for the sum of arithmetic sequence
𝑛
𝑆𝑛 = 2 [2𝑎1 + (𝑛 − 1)𝑑 ], substitute Sn = 116, a1 = –3 and d = 5.
We have
𝑛
116 = [2(−3) + (𝑛 − 1)5]
2
𝑛
116 = [2(−3) + 5𝑛 − 5]
2

25
𝑛
116 = [−6 + 5𝑛 − 5]
2
𝑛
116 = [5𝑛 − 11]
2
𝑛
2 [116 = (5𝑛 − 11)]
2
232 = 𝑛(5𝑛 − 11)
232 = 5𝑛2 − 11𝑛
5𝑛2 − 11𝑛 − 232 = 0

Using quadratic formula, we have,


a = 5; b = –11; c = –232
−𝑏 ± √𝑏2 − 4𝑎𝑐
𝑛=
2𝑎
−(−11) ± √(−11)2 − 4(5)(−232)
𝑛=
2(5)
11 ± √121 + 4640
𝑛=
10
11 ± √4761
𝑛=
10
11 ± 69
𝑛=
10
Since we are looking for the number of terms n, the only accepted
solution is the positive solution. That is
𝒏=𝟖

Therefore, eight (8) terms of the sequence –3, 2, 7, … is needed to


have a sum of 116.

Illustrative Example 3:
Find the sum of the first 40 terms of the arithmetic sequence whose
first and third terms are 15 and 21, respectively.
Solution:
We need to solve first for d by substituting a1 = 15, a3 = 21 and n = 3
to the formula
𝑎𝑛 = 𝑎1 + (𝑛 − 1)𝑑
21 = 15 + (3 − 1)𝑑
21 = 15 + 2𝑑
6 = 2𝑑
𝑑=3

Solving for S40, substitute a1 = 15, n = 40 and d = 3 to the formula


𝑛
𝑆𝑛 = [2𝑎1 + (𝑛 − 1)𝑑 ]
2
40
𝑆40 = [2(15) + (40 − 1)3]
2
𝑆40 = 20[30 + 117]
26
𝑺𝟒𝟎 = 𝟐𝟗𝟒𝟎

Therefore, the sum of the first 40 terms is 2940.

2. Analysis

Is it possible to find the sum of terms of an arithmetic sequence?


If it is possible to find its sum, how did you obtain the sum of the
arithmetic sequence in the activity?
If the sequence contains large number of terms in the arithmetic
sequence, is it reasonable to use the previous solution that you have
used?
How to get the sum of terms in an arithmetic sequence?

3. Guided Practice

Supply each blank by a correct answer following the task at the right to
solve the problem.

a. Find the sum of the first 15 terms of the arithmetic sequence 9, 12,
15, …
Given: a1 = ____ ; d = ____ ; n = ____
Solution:
Solve for a15
𝑎𝑛 = 𝑎1 + (𝑛 − 1)𝑑
𝑎𝑛 = ___ + (___ − 1)___ substitute a1, n and d
𝑎𝑛 = 9 + (____)3 subtract the terms inside the parenthesis
𝑎𝑛 = 9 + (____) multiply
𝑎𝑛 = _____ add
Then solve for S15.
𝑛
𝑆𝑛 = (𝑎1 + 𝑎𝑛 )
2
___
𝑆15 = (____ + ____) substitute n, a1 and a15
2
15
𝑆15 = (_____) add the terms inside the parenthesis
2

_____
𝑆15 = 2 find the product of the numerator
𝑆15 = ______ divide

b. Find the sum of the first 10 terms of the arithmetic sequence whose
a1 and a4 are 5 and 38, respectively.
Given: a1 = ____ ; a4 = ____ ; n = ____
Solution:
27
𝑎𝑛 = 𝑎1 + (𝑛 − 1)𝑑
____ = _____ + (____ − 1)𝑑 substitute the given
38 = 5 + (____)𝑑 subtract the terms inside the
parenthesis
____ = 3𝑑 apply APE
𝑑 = ____ apply MPE

Solve for S10.


𝑛
𝑆𝑛 = [2𝑎1 + (𝑛 − 1)𝑑 ]
2
___
𝑆𝑛 = [2(___) + (____ − 1)____] substitute a1, n and d
2
10
𝑆𝑛 = [____ + (_____)11] multiply 2 and a1 and then
2
subtract the value of n and 1
10
𝑆𝑛 = [10 + ____ ] multiply
2
10
𝑆𝑛 = 2 [____ ] add
𝑆𝑛 = ____[109] divide
𝑆𝑛 = _______ multiply

4. Independent Practice

Find Sn for each of the following given.


1. 6, 11, 16, 21, 26, 31, 36, 41, 46; S9
2. 10, 15, 20, 25, …; S20
3. a1 = 25, d = 4; S12
4. a1 = 65, a10 = 101; S10
5. a4 = 41, a12 = 105; S8

5. Generalization

The sum of terms in an arithmetic sequence can be solve using the


𝒏
formula 𝑺𝒏 = 𝟐 (𝒂𝟏 + 𝒂𝒏 ), given the 1st and last term of the sequence or
𝒏
𝑺𝒏 = 𝟐 [𝟐𝒂𝟏 + (𝒏 − 𝟏)𝒅], given the first term and the common
difference.

6. Application

Answer the following problems.


1. Find the seating capacity of a movie house with 40 rows of seats if
there are15 seats on the first row, 18 seats in the second row, 21
seats in the third row and so on.
28
2. A store sells Php 1000 worth of Suman sa Kawit, a delicacy from
Kawit, Cavite, during its first week. The owner of the store has set a
goal of increasing her weekly sales by Php 300 each week. If we
assume that the goal is met, find the total sales of the store during
the first 15 weekof operation.
3. Francisco plans to save Php 10 every week on his Bamboo coin
bank. If he will increase his savings by Php 1.50 every succeeding
week, how many weeks is needed to save a total amount of Php
219?

7. Assessment

Each row of the table contains the values of three quantities a 1, d, an, or
Sn of an arithmetic sequence. Complete the table below by solving the
other two.

a1 d an n Sn
1. 2 5 10
2. 7 –2 –15
3. –1.5 15 20
4. 5 7 365
5. 7 8 64

IV - ASSIGNMENT
1. Follow-up
a. Find the sum of all odd numbers from 1 to 99.

2. Study: Geometric Sequence


a. Define geometric sequence and common ratio.
b. Identify which of the following is NOT an arithmetic sequence.
How does the non-arithmetic sequence was formed? Identify its
pattern.
1, 4, 7, 10, … 9, 12, 15, 18, …
2, -10, 50, -250, … 5, 10, 20, 40, …
2, 6, 10, 14, … 3, 12, 48, 192, …
7, 12, 17, 22, … 4, 11, 18, 25, …
1, 3, 9, 27, … 1, 4, 16, 64, …

Electronic Sources: https://www.mathsisfun.com/algebra/sequences-sums-arithmetic.html


http://www.purplemath.com/modules/series4.htm
http://www.analyzemath.com/math_problems/arith-seq-problems.html

29
Answer Key:

Guided Practice
Supply each blank by a correct answer following the task at the right to solve the problem.

a. Find the sum of the first 15 terms of the arithmetic sequence 9, 12, 15, …
Given: a1 = 9 ; d = 3 ; n = 15
Solution:
Solve for a15
𝑎𝑛 = 𝑎1 + (𝑛 − 1)𝑑
𝑎𝑛 = 9 + (15 − 1)3 substitute a1, n and d
𝑎𝑛 = 9 + (14)3 subtract the terms inside the parenthesis
𝑎𝑛 = 9 + 42 multiply
𝑎𝑛 = 51 add
Then solve for S15.
𝑛
𝑆𝑛 = (𝑎1 + 𝑎𝑛 )
2
15
𝑆15 = 2 (9 + 51) substitute n, a1 and a15
15
𝑆15 = (60) add the terms inside the parenthesis
2
900
𝑆15 = 2 find the product of the numerator
𝑆15 = 450 divide

b. Find the sum of the first 10 terms of the arithmetic sequence whose a 1 and a4
are 5 and 38, respectively.
Given: a1 = 5 ; a4 = 38 ; n = 4
Solution:
𝑎𝑛 = 𝑎1 + (𝑛 − 1)𝑑
38 = 5 + (4 − 1)𝑑 substitute the given
38 = 5 + (3)𝑑 subtract the terms inside the parenthesis
33 = 3𝑑 apply APE
𝑑 = 11 apply MPE

Solve for S10.


𝑛
𝑆𝑛 = [2𝑎1 + (𝑛 − 1)𝑑 ]
2
___
𝑆𝑛 = 2 [2(5) + (10 − 1)11] substitute a1, n and d
10
𝑆𝑛 = [10 + (9)11] multiply 2 and a1 and then subtract the
2
value of n and 1
10
𝑆𝑛 = [10 + 99 ] multiply
2
10
𝑆𝑛 = [109] add
2
𝑆𝑛 = 5[109] divide
𝑆𝑛 = 545 multiply
30
Independent Practice
Find Sn for each of the following given.
1. 234
2. 1150
3. 564
4. 830
5. 360

Application
Answer the following problems.
1. 2940 seats
2. Php 35250
3. 12 weeks

Assessment
Each row of the table contains the values of three quantities a 1, d, an, or Sn of an
arithmetic sequence. Complete the table below by solving the other two.
1. an = 47; Sn = 245
2. n = 12; Sn = – 48
3. a1 = 43.5; Sn = 585
4. n = 10; an = 68
5. a1 = 9; d = - 2/7

31
Module 1: Sequences
Lesson 2: Geometric and other sequences
Learning Competency 5: Differentiate a geometric sequence from an arithmetic sequence

I - OBJECTIVES
a) Recognize which is an arithmetic sequence and geometric sequence
b) Identify the difference between arithmetic sequence and geometric sequence,
c) State whether each of the following sequences is arithmetic or geometric:
II - SUBJECT MATTER

Topic: Geometric Sequence


Sub-topic: Differentiate a geometric sequence from an arithmetic sequence
Materials : Practice exercices
Reference:Mathematics Learner’s Module, pp. 28 and
Mathematics Teacher’s Guide, p. 24
III - PROCEDURE
A. Preliminaries:
Do you remember the sitting arrangement done last year when you took the
NCAE. There were 30 students in each room. The table 1 shows that the number of
students varies directly as the number of rooms or as the number of rooms
increases, the number of students also increases. Can you guess the number of
students when there are 12 rooms used? Table 1 is an example of Arithmetic
Sequence.

Table 1
No. of Rooms 1 2 3 4 5 6
No. of Students 30 60 90 120 150 180

Suppose that the number of a certain bacteria grows as shown in table 2


below. At the start, there are only 1, 000 bacteria and after 1 hour the number of
bacteria is doubled. It is consistent that based from the observation, the number of
bacteria is always doubled every hour. Can you tell the number of bacteria after 7
hours? 10 hours? Table 2 is an example of Geometric Sequence.

Table 2
No. of Hours 0 1 2 3 4
No. of Bacteria 1, 000 2, 000 4, 000 8, 000 16, 000

32
B. Lesson proper
1. Teaching /Modeling
A. Facts observed in table 1:
1. The sequence of the number of students is 30, 60, 90, 120, 150, 180
2. This is an arithmetic sequence
3. The first term is increased by 30 to get the second term, and the second term
is increased by 30 to get the third term, and so forth and so on.

B. Facts observed in table 2:


1. The sequence of the number of bacteria is 1000, 2000, 4000, 8000, 16000
2. This is a geometric sequense
3. The first term is multiplied by 2 to get the second term, and the second term
is multiplied by 2 to get the third term, and so on.
Guide Questions:
1. What have you observed about the differences of the arithmetic and geometric
sequence?
2. Can you tell the number of students who took the NCAE if there were 12 rooms used
during the examination?
3. How many bacteria would there be after 7 hours if we consider the data in table 2?
4. Can you define arithmetic sequence? geometric sequence?
Developmental Act
Let us do the activity below.

Activity 1: State whether each of the following sequences is arithmetic or geometric:


1) 3, 7, 11, 15, 19, 23 …
2) 2, 6, 18, 54, 162 …
3) 7, 14, 28, 56, 112 …
4) 5, 20, 80, 320 …
5) 7√2, 5√2, 3√2, √2 …

2. Analysis:
1. How do you find doing the activity?
2. Which of the items are arithmetic sequence and geometric sequence?
3. What is the most important characteristics that you should remember in
identifying arithmetic or geometric sequence?
4. What kind of sequence is 3, 7, 11, 15, 19, 23 …?
5. What kind of sequence is 2, 6, 18, 54, 162 …?

3. Guided Practice:
Example 1. Examine the sequence 12, 17, 22, 27, 32, …

33
Step 1. Subtract the second term by the first term
d = 17 – 12 = 5 ( d = right term – left term)
Step 2. Check if the difference between the third term and the second term is the
same with step 1.
d = 22 – 17 = 5 ( d = right term – left term)
Step 3. Therefore, the sequence 12, 17, 22, 27, 32, … has a common difference,

Example 2. Examine the sequence 2, 6, 18, 54, 162 …


Step 1. Divide the second term by the first term
6
r = = 3 (r = right term divided by the left term)
2
Step 2. Check the result if the same operation is applicable to get the third term.
18
r= = 3 (r = right term divided by the left term)
6
Step 3. Therefore, the sequence 2, 6, 18, 54, 162 …has a common ratio, r

4. Independent Practice
State whether each of the following sequences is arithmetic or geometric:
1) 3, 9, 27, 81, 243 …
2) 7, 21, 63, 189, 567 …
3) 7, 14, 21, 28, 35 …
4) 5, 25, 45, 65, 85 …
5) 2, 8, 32, 128, 512…
5. Generalization
Arithmetic Sequence is a sequence where each term after the first is obtained
by adding the same constant, called the common difference.
Common Difference is a constant addrd to each term of an arithmetic sequence
to obtain the next term of the sequence.
Geometric Sequence is a sequence where each term after the first is obtained
by multiplying the preceeding term by a nonzero constant called the common
ratio.
Common Ratio is a constant multiplied to each term of a geometric sequence to
obtain the next term of the sequence.

6. Application
State whether each of the following sequences is arithmetic or geometric:
1. 4, 12, 36, 108, 324…
2. -4, 13,, 30, 47, 64…
3. 3, -6, 12, -24, -72 …
4. 3, 5, 7, 9, 11…
5. -3, 3, -3, 3, -3…

34
7. Assessment
Answer the following:
1) Define arithmetic sequence and geometric sequence.
2) What is common difference? Common ratio?

State whether the given sequence is an arithmetic or geometric


3) 77, 70, 63, 56, 49 …
4) 6400, -1600, 400, -100, 25 …
5) 6, 30, 150, 750, 3750 …

I. ASSIGNMENT
Answer Activity 3 of Module 2, lesson 2 . Mathematics Learners Module p 28

35
Answer Key:
Independent Practice/ I can do this
1. Geometric sequence
2. Geometric sequence
3. Arithmetic sequence
4. Arithmetic sequence
5. Geometric sequence

Application/Let’s do more
1. Geometric sequence
2. Arithmetic sequence
3. Geometric sequence
4. Arithmetic sequence
5. Geometric sequence

Assessment/Challenge Yourself
1. Arithmetic Sequence is a sequence where each term after the first is obtained
by adding the same constant, called the common difference.
Geometric Sequence is a sequence where each term after the first is obtained by
multiplying the preceeding term by a nonzero constant called the common ratio.
2. Common Ratio is a constant multiplied to each term of a geometric sequence to
obtain the next term of the sequence.
Common Difference is a constant addrd to each term of an arithmetic sequence
to obtain the next term of the sequence.
3. Arithmetic sequence
4. Geometric sequence
5. Geometric sequence

36
Module 1: Sequences
Lesson 2: Geometric and other sequences
Learning Competency 6: Determine the nth of a given geometric sequence

I - OBJECTIVES

b) Solve the common ratio of the geometric sequence;


b) Find the nth term in a geometric sequence using the common ratio, and
c) Appreciate the use of geometric sequence formula in solving real-life problems.
II - SUBJECT MATTER

Topic: Geometric Sequence


Sub-topic: Determine the nth Terms of a Geometric Sequence
Materials : Practice exercices
Reference:Mathematics Learner’s Module, pp. 26-30 and
Mathematics Teacher’s Guide, p. 24
III - PROCEDURE
A. Preliminaries
All of us, young and old, boys and girls are delighted with the amazing help brought
by cellphones in our lives. We text our friends many times everyday. Have you ever counted
the number of texts you send everyday? Do you receive text that tells you to send it also to
two of you friends? How many persons will receive that message after the 7 rounds of
chained messages? This situation is an example of Geometric Sequence.
This lesson will help you find solution to problems which involve geometric
sequence.
B. Lesson proper
1. Teaching /Modeling
Activity 1. Paper folding: Give ½ sheet of graphing paper to each student. (Let the
students perform the activity below and tell them to record the results in the provided table)
Number of Folds 0 1 2 3
Number of squares

1. Count the squares in a ½ sheet of graphing paper.


2. Fold the graph paper into two equal parts. How many squares does one side
have?
3. Fold again the graph paper into equal parts and again, count the number of
squares.
37
4. Repeat the process three times and record the results in the table above.
Guide Questions:
1. How many squares does a ½ sheet of graphing paper have at the start of the activity?
2. How many squares after the first fold (one side only)? After the second fold (one side
only)? After the third fold (one side only)?
3. Is there a pattern in the number of squares obtained after three folds?
4. Can you guess the number of squares of a side after the fourth fold?
5. Can you make an equation or formula based from that activity?
Developmental Act
To solve problem about geometric sequence, you must understand first the
concept of common ratio. The exercises below will help you conceptualize the
meaning of a ratio ( r ).

Activity 2: Find the ratio of the second number to the first


1) 3, 27
2) –3, -81
3) 4, 16
4) 5, 40
5) 7, 112
6) 6, 162

Activity 3: Terminologies in the geometric sequence


Other Ways to Write the Terms
Term In Factored Form In Exponential Form
a1 = 2 2 2 x 50
a2 = 10 2x5 2 x 51
a3 = 50 2x5x5 2 x 52
a4 = 250 2x5x5x5 2 x 53
a5 = 1250 2x5x5x5x5 2 x 54
: : :
an ?

2. Analysis:
1. Look at the two ways of writing the terms. What does 2 represent?
38
2. For any two consecutive terms, what does 5 represent?
3. What is the relationship between the exponent of 3 and the position of the
term?
4. If the position of the term is n, what must be the exponent of 5?
5. What is an for this sequence?
6. In general, if the first term of a geometric sequence is a 1 and the common ratio
is r , what is the nth term of the sequence?

Hint: Use the formula to determine the nth term of the geometric sequence.
an = a1rn -1 , Geometric Sequence is a sequence where each term after the first is obtained
by multiplying the preceeding term by a nonzero constant called the common ratio.
Where;
an - Last term
a1 - First term
r - Common ratio
n - Number of terms (or position of the term)
3. Guided Practice:
Example 1. Find the missing terms in the geometric sequence 6, 12, 24, __, __
Solution:
Step 1. Divide the second term by the first term to find the common ratio, 12 ÷ 6 = 2,
therefore the common ratio, r = 2.
Step 2. Multiply the third term by the common ratio to find the fourth term, 24 x 2 = 48
Step 3. Multiply the fourth term by the common ratio to find the fifth term, 48 x 2 = 96
Answer: The missing terms are 48 and 96

Example 2. Find the 7th term in the geometric sequence 7, 28, __, 448, __, __,7th
Solution:
Step 1: Find the common ratio, r
r = term at the right divided by the term at the left
r = 28 ÷ 7
r=4
Step 2: Use the formula an = a1rn -1 to find the last term or the 7th term.
an = a1rn -1
a7 = 7(4)7 – 1
a7 = 7(4)6
a7 = 7(4,096)
39
a7 = 28,672 (last term or the 7th term)
1. Independent Practice
A. Supply the missing terms in the given geometric sequence:
1) 3, __, 48, __, 192, 768, __
2) 8, 24, __, __, __, __
3) __, __, 4, 1, __, __
4) 2, -6, 18, __, __, __
1 3
5) , , , ,
2 4

B. Find the nth term of the given geometric sequence below:


1) 2, 6, 18, …8th term
2) 5, 20, 80, … 11th term
3) 9, 54, … 7th term
4) __, __, 12, 48, … 12th term
5) __, 18, __, 72, … 13th term

2. Generalization
an = a1rn -1 , Geometric Sequence is a sequence where each term after the first is
obtained by multiplying the preceeding term by a nonzero constant called the
common ratio. Where;
an - Last term
a1 - First term
r - Common ratio
n - Number of terms (or position of the term)

3. Application
1. In the given table below, how many will be the Boy Scouts of GEANHS on
SY 2019 – 2020?

Gen. E. Aguinaldo National High school (GEANHS)


Period Number of Boy Scouts
SY 2012 - 2013 17
SY 2013 - 2014 25
SY 2014 - 2015 35
SY 2015 - 2016 49
SY 2016 - 2017 69
⁞ ⁞
SY 2019 - 2020 ?

40
8 8
2. What is the common ratio in the geometric sequence 24, 8, ,
3 9
3. What is the last number in this geometric sequence 7, 21, … 5th?
4. What are the missing terms in the sequence 4, __, __, 108, __ ?
5. What is the last term in the geometrc sequence 16, 8, 4 … 8th ?

4. Assessment

1. Find the missing terms in the geometric sequence -2, __, __, -16, -32, -64.
2. What is the common ratio in the geometric sequence 3, -12, 48, -192, 768 ?
3. What is the last term in this geometric sequence 2, 14, …7th ?
4. What is the 10th term of the geometric sequence -3, 15, -75… ?
5. What is the 13th term in the sequence __, 4, __, 16, __, __, -128, …?

IV. ASSIGNMENT

1. Once upon a time, there was a king who madly fall inlove with a beautiful lady.
The king was so determined to have the lady be his wife. On the contrary, the
lady seemed not feeling the same way as the king. The lady who is not only
confidently beautiful with a heart but also endowed with high level of
intelligence gave challenge to the king. “If you can fill the squares of the chess
board with grains of rice starting one grain in the first square and doubled in
the succeeding squares, then I will accept your proposal, said the lady”.
a) How many grains of rice would there be at the fifth squares? 27th
square?
b) How about at the last square? How many grains of rice were there at
the last square/64th square?
c) What do you think would happen to the king’s proposal? Was he able
to succeed the challenge?
2. The SSG officers are advocating for a tree planting project starting June 2016
to March 2017. Each officer is given the task to plant 1 seedling on June, 2
seedlings on July, and 4 seedlings on August. How many seedlings have been
already planted by each officer on March 2017?
3. The GEANHS is conducting OPLAN LINIS to diminish if can not totally
eradicate the mosquitos that bring dengue. Dengue is a deadly virus because
its number is doubled every day. Suppose there are 500 starting virus in the
body of the sick, how many virus will there be after one week?

41
Answer Key:
Independent Practice/ I can do this
A. Supply the missing terms Assignment

1. 3, 12, 48, 192, 768, 3072 1)


2. 8, 24, 72, 216, 648, 1944 a) 5th square has16 grains
1 1 th
3. 64, 16, 4, 1, , 27 square has 67,108,864 grains
4 16
a) 2, -6, 18, -54, 162, -486 b) 9.22x1018
4.
1 3 9 27 81
5. , , , , c) let the students give their opinions
2 4 8 16 32

B. Find the nth term 2) 2,408


1. 4,374 3) 32,000
2. 5,242,880
3. 419,904
4. 3,145,728
5. 36,864

Application/Let’s do more
1. 169
1
2.
3
3. 567
4. 4, 12, 36, 108, 324
1
5.
8

Assessment/Challenge Yourself
1. -4 and -8
2. -4
3. 235,298
4. 5,859,375
5. 8,192

42
Module 1: Sequence
Lesson 2: Geometric Sequence
Learning Competency 7: Finding the common ratio given the first and last terms

I - OBJECTIVES

a. Solve the common ratio when two consecutive terms are given,
b. Find the common ratio when the first and last terms are given, and
c. Appreciate the use of the common ratio in solving geometric sequence.
II - SUBJECT MATTER

Topic: Geometric Sequence


Sub-topic: Geometric Means
Materials : Practice exercices
Reference: Mathematics Learner’s Module, p. 30 and
Mathematics Teacher’s Guide, p. 24
III - PROCEDURE
A. Preliminaries
We learned from our past lesson the procedure on how to find the common ratio
when two consecutive terms are given.
Let’s take a look once again on how to find the common ratio by solving the problem
below:
Example: Find the common ratio of the geometric sequence 4, 12, 36, 108
Step 1: Get two consecutive terms. 4 and 12
Step 2: Divide the 2nd term by the 2nd term to get the common ratio.
12
Common ratio =
4
r=3

Try using the 3rd term and the 2nd term, or divide the 3rd term by the 2nd term, you will
get the same answer as we have at the example above.
B. Lesson proper
1. Teaching /Modeling
Activity 1. Solve the common ratio in the geometric sequence 4, __, __, __, 64

Step 1: Identify the first term, last term and the number of terms in the problem.

43
a1 = 4
a5 = 64
n=5
Step 2: Use the formula, an = a1rn -1 which we learned from our past lesson to find
the common ratio.

an = a1rn -1
64 = 4r5-1
64 = 4r4
64 44
= r
4 4
16 = r4
2 4 = r4
±2 = r
Answer: The common ratios, r = 2 and r = -2

2. Analysis: Questions below will serve as your guide in finding the common ratio of
the geometric sequence, especially when the first term, last term and the number
of terms are given:

1. What are the three properties of the geometric sequence that we need to know
in order to solve the common ratio?
2. What is the formula to be used to solve the common ratio?
1
3. Why do we have to multiply both sides of the exponential equation 64 = 4r 4 by ?
4
4. What do we do to number 16? Notice that 16 = r4 becomes 24 = r4 .
5. Why do we cancel the exponents in the equation 24 = r4
3. Guided Practice:
Example: Find the common ratio in the sequence 8, __, __, __, 128.
Solution:
Step 1: an = a1rn -1 Use this formula to find the common ratio.
128 = 8r5 – 1 Substitute the first term, last term and the exponent
128 = 8r4 Simplify the exponent
128 8 4
= r Apply Multiplication Property of Equality to cancel the coefficient of r
8 8
16 = r4 Make the coefficient of r equal to 1
4 4
2 =r express 16 in exponential form such that the exponential equation has
the same exponents
24 = r4 espression with the same exponents are equal
±2 = r (1) if the exponent being cancelled is even, there are positive and
negative roots, and

44
(2) if the exponent being cancelled is odd, there is either positive or
negative common ratio.

Answers: ±2 there are two common ratios

4. Independent Practice

Give the common ratio in each of the following geometric sequences:


1) 8, __, __, __, 5000
2) 3, __, __, 648
3) 7, __, __, __, __, 1701
4) 11, __, __, __, __, __, 704
5) 4, __, __, 1372

5. Generalization
a. Determine the number of terms, first term and last term in the given geometric
sequence.
b. Use this formula to find the common ratio.
c. Substitute the first term, last term and the exponent
d. Simplify the exponent
e. Apply Multiplication Property of Equality to cancel the coefficient of r or make
the coefficient of r equal to 1
f. Express both sides ot the exponential equation with the same exponent
g. Cancell the exponent, since expressions with the same exponents are equal
h. If the exponent being cancelled is even, there are two roots which are positive
and negative roots, and
i. If the exponent being cancelled is odd, there is only one root/common ratio
and that is either positive or negative.

6. Application

Answer the following:


1. What is the common ratio if three geometric means are inserted between 7 and
567?
2. What is the common ratio in the geometric sequence 4, __, __, 108
3. 5 geometric means are inserted between 6 and 4,374, find the common ratio.
4. If six terms are to be inserted between 8,748 and 4 being the first term and last
term respectively, What is the common ratio?
5. The growth rate of ants is rapidly increasing. There were 10 ants at the
beginning but on the 7th day, it was counted by keizelyn and she found out that
the total number of ants was already 640. Make a table to show the number of
ants from first day to seventh day.

45
7. Assessment:

Find the common ratio of each of the following geometric sequence:


1) 5, __, __, __, __, 160
2) 3, __, __, __, -5,625
3) 256, __, __, 4
4) 8, __, __, __, 648
2
5) 2, __, __, __, __,
243

IV - ASSIGNMENT
Find the common ratio of the following geometric sequences (Show your solution).
a) -2, __, __, __,__, -64.
b) 2,__, __,__, __, __, 1458?
c) 1782, __, __, __, 22
16
d) 4 geometric means between 3 to
27
e) 4 geometric means are inserted between 3, -3072

46
Answer Key:
Independent Practice/ I can do this

1. ±5
2. 6
3. 3
4. ±2
5. 7
Application/Let’s do more
1) ±3
2) 23
3) ±3
1
4)
3
5) ±2

Assessment/Challenge Yourself
6. 2
7. -5
1
8.
4
9. ±3
1
10.
3

47
Module 1: Sequence
Lesson 2: Geometric Sequence
Learning Competency 8: Determine the geometric means between terms of a
geometric sequence

I - OBJECTIVES

a. Find the geometric means of a geometric sequence,


b. Use the common ratio to find the geometric means between two terms
c. Appreciate the use of geometric sequence formula in solving real-life problems.
II - SUBJECT MATTER

Topic: Geometric Sequence


Sub-topic: Geometric Means
Materials : Practice exercices
Reference: Mathematics Learner’s Module, p. 30 and
Mathematics Teacher’s Guide, p. 24
III - PROCEDURE
A. Preliminaries
From Bailen, what are the barangays that you will pass by if you are going to
Alfonso? How many barangays are there inside the two endpoints/towns? There are four
barangays. What are those?
Answer: Cast. Cerca, Cast. Lejos, Marahan, Alas-as. Bailen and Alfonso are
endpoints.
The number of barangays mentioned above has a similarity to the concept of our
topic today. There are four (4) barangays that you will pass by when you started from Bailen
going to Alfonso.
B. Lesson proper
1. Teaching /Modeling
Activity 1. Illustrate the geometric means in the geometric sequence 4, 8, 16, 32, 64
1. What is the first term, a1 ?
2. What is the last term, an ?
3. Can you guess the geometric means?
4. How many terms are inserted between 4 and 64?

Activity 2. Finding the geometric means


Let us use the geometric sequence in Activity 1, ( 4, 8, 16, 32, 64 ). For instance, the
only given in the problem are the first term and the last term, as in 4, __, __, __, 64. How
can we solve the geometric means?
48
Solution:
Step 1: Find the common ratio

an = a1rn -1
64 = 4r5-1 substitute the first and last terms
4
64 = 4r simplify the exponent
64 4 4
= r apply MPE
4 4
4
16 = r coefficient of r now is 1
4 4
2 =r exponential equation
±2 = r common ratio
Step 2: Multiply the first term by the common ratio r = 2 to get the second term.
Repeat the process until you solve the three geometric means. Use r = -2 to
find the other geometric sequence
a2 = 4 x 2 ; a2 = 8
a3 = 8 x 2 ; a3 = 16
a4 = 16 x 2 ; a4 = 32
Note: There are two common ratios, therefore there are also two sets of geometric
sequences: 4, 8, 16, 32, 64 and 4, -8, 16, -32, 64

Answer: The three geometric means are 8, 16, 32 and -8, 16, -32

2. Analysis:
1. How many terms are there in the geometric sequence including the first and
last terms?
2. What do you need to solve first to find the geometric means of the given
geometric sequence?
3. What is your conjecture if n in rn is odd? even?
4. Given the first term and the common ratio, how can you find the second term?
5. How can you find the third term?
3. Guided Practice:
Example: Insert two geometric means in the sequence 6, __, __, 384.
Solution:
Step 1: Determine the number of terms
There are four terms in the given geometric sequence, n = 4

Step 2: Find the common ratio (r).


an = a1rn -1 Use this formula to find the common ratio.
384 = 6r4 – 1 Substitute the first term, last term and the exponent
384 = 6r3 Simplify the exponent
49
384 6 4
= r Apply Multiplication Property of Equality to cancel the coefficient of r
6 6
64 = r3 Make the coefficient of r equal to 1
43 = r3 Express 16 in exponential form such that the exponential equation
has the same exponents
3 3
4 =r Expression with the same exponents are equal
4 =r (1) if the exponent being cancelled is even, there are positive and
negative common ratios, and
(2) if the exponent being cancelled is odd, there is either a positive
or a negative common ratio.
Step 3: Use r = 4, Multiply the first term by the common ratio to find the second term.
Answers: 6, 24, 86, 384 (the geometric means are 24 and 86)

4. Independent Practice
Give the geometric means of the following geometric sequence:
1) 3, __, __, __, 1875
2) 6, __, __, 2058
3) 8, __, __, __, __, 1944
4) 1, __, __, 1331
5) 224, __, __, __, __, 7

5. Generalization
To find the geometric means of the given geometric sequence
a) Identify the number of terms in a geometric sequence (including the geometric
means, the first term and the last term).
b) Solve the common ratio.
c) Multiply the first term by the common ratio to find the second term.
d) Multiply the second term by the common ratio to find the third term, and repeat
the procedure until you solve the required geometric means.
e) In bx = rn , If the exponent being cancelled is odd, there is only one common
ratio and that is either positive or negative; while if n is even, there are two
common ratios which are positive and negative.

6. Application

Answer the following


1. What are the three geometric means between 3 and 768?
2. What are the missing terms in the sequence 5, __, __, 320
3. Insert 5 geometric means between 6 and 4,374
4. What are the two terms between 1024 being the first term and 2 as the last.
5. The number of a certain bacteria is doubled every hour. If the initial number of
bacteria is 800 units and becomes 25,600 on the 6th day, how many bacteria are
there on the third day?

50
6. Assessment
Find the geometric means of the following geometric sequences (Show your
solution)

1. Find the missing terms in the geometric sequence -2, __, __, __,__, -64.
2. Insert 4 geometric means between 3, -3072 ?
3. What are the 5 geometric means in the sequence 2,__, __,__, __, __, 1458?
4. Complete the geometric sequence 1782, __, __, __, 22
16
5. Insert 3 geometric means from 3 to .
27

IV. ASSIGNMENT:
Answer the following problems about Geometric Means:
1. What are the four geometric means between -4 and 972?
2. What are the missing terms in the sequence 5120, __, __,__, __, 5
3. Insert 5 geometric means between 7 and 28,672
4. What are the three terms between 160 being the first term and 10 as the last.
5. Insert five geometric means to the geometric sequence 4374, __, __, __, __, __, 6

51
Answer Key:
Independent Practice/ I can do this

1. 15, 75, 375 and -15, 75, -375


2. 42, 294
3. 24, 72, 216, 648
4. 11, 121
5. 112, 56, 28, 14

Application/Let’s do more
3. 12, 48, 192 and -12, 48, -192
4. 20, 80
5. 18, 54, 162, 486, 1458 and -18, 54, -162, 486, -1458
6. 128, 16, 2
7. 3200 units

Assessment/Challenge Yourself
11. -4, -8, -16, -32
12. -12, 48, -192, 768
13. 6, 18, 54, 162, 486
14. 594, 198, 66
4 8
15. 2, ,
3 9

Assignment:
1) 12, -36, 108, -324
2) 1280, 320, 80, 20
3) 28, 112, 448, 1792, 7168 and -28, 112, -448, 1792, -7168
4) 80, 40, 20
5) 1458, 486, 162, 54, 18 and -1458, 486, -162, 54, -18

52
Module 1: SEQUENCES
Lesson 2: Geometric and Other Sequences
Lesson 2.1: Finite Geometric Sequence
Learning Competency 9: Finds the sum of the terms of a given finite geometric sequence.

I - OBJECTIVES
a. Determine the sum of the first n-terms of a given finite geometric sequence.
b. Apply the concepts of finite geometric series in real life situations.
c. Develop analytical mind while applying the lesson in real life situation.
II - SUBJECT MATTER
Topic: Geometric Sequence
Sub-topic: Sum of Finite Geometric Sequence
Materials: Activity Sheets
References: TG pages 22-25
LM pages 31-34
Intermediate Algebra by Pastor B. Malaborbor, et.al, pages
317-321
e-math Intermediate Algebra by Orlando A. Oronce, et.al,
pages 444-450
III - PROCEDURE
A. Preliminaries
Motivation
(QUICK THINKING ONLY!)
Gabriel wants to spend his 12-day Christmas break productively by
working in “Alamat Restaurant” located at Picnic Grove. The manager offers 2
salary scheme: Php100 per day or Php1 on the 1st day, Php2 on the 2nd day,
Php4 on the 3rd day and double the salary each day until the 12th day.
If you were Gabriel, which salary scheme are you going to accept?
Why?
B. Lesson Proper
1. Teaching/Modeling
From the situation above, how much will Gabriel receive in the
first salary offer after 12 days?
Make a table using the salary of Gabriel in each day.
Day 1 2 3 4 5 6 7 8 9 10 11 12
Salar 1 2 4 8 1 3 6 12 25 51 102 204
y 6 2 4 8 6 2 4 8
After 12 days, how much will Gabriel receive?
Aside from adding all the salaries each day which is equal to
4095, other solution is by using the formula to find the sum of a
Geometric Sequence when r ≠ 1 or -1 is
Sn = (a1 - anr) / (1 - r) or
53
Sn = [a1(1 - rn)] / (1 – r)
Solution: Since r = 2; n = 12; a1 = 1
S12 = [1 ( 1– 212)] /(1-2)
= 4095
To get the sum of a finite geometric sequence when:
Case 1: r = 1
Example: 5, 5, 5, 5, 5, 5, 5, 5
Solution: Sn = a1(n)
= 5(80) = 40
Case 2: r = -1 and n is even
Example: 7, -7, 7, -7, 7, 7
Solution: Sn = 0
Case 3: r = -1 and n is odd
Example: 7, -7, 7, -7, 7, -7, 7
Solution: Sn = a1
=7
2. Analysis
1. What is the formula to find the sum of the terms in a finite
Geometric sequence?
2. If you will use the first formula, what are the data that you need?
What about in the second formula?
3. Is there another way to get the correct answer? Explain briefly your
solution.

3. Guided Practice
Do the following with a partner!
In text brigade relay scheme of Grade-X May Kusa, the following
are the number of receivers of the text after the third transmittal. Find
the total number of person who received the text after the 6 th
transmittal, assuming that the relay is not broken and each message is
successfully transmitted.
1. 4, 12, 36, . .
Solution:
Determine the values of a1, n and r.
a1 = ____; n = ______; r = _______
Write the formula in finding the sum of finite
Geometric sequence.
Sn = ____________________
Substitute the values of a1, n and r.
Sn = ____________________
Simplify.
Sn = ____________________

54
2. 3, 6, 12, . . .
Solution:
Determine the values of a1, n and r.
a1 = ____; n = ______; r = _______
Write the formula in finding the sum of finite
Geometric sequence.
Sn = ____________________
Substitute the values of a1, n and r.
Sn = ____________________
Simplify.
Sn = ____________________
3. 2, 4, 8, . . .
Solution:
Determine the values of a1, n and r.
a1 = ____; n = ______; r = _______
Write the formula in finding the sum of finite
Geometric sequence.
Sn = ____________________
Substitute the values of a1, n and r.
Sn = ____________________
Simplify.
Sn = ____________________
4. 4, 24, 144, . . .
Solution:
Determine the values of a1, n and r.
a1 = ____; n = ______; r = _______
Write the formula in finding the sum of finite
Geometric sequence.
Sn = ____________________
Substitute the values of a1, n and r.
Sn = ____________________
Simplify.
Sn = ____________________
5. 4, 16, 64, . . .
Solution:
Determine the values of a1, n and r.
a1 = ____; n = ______; r = _______
Write the formula in finding the sum of finite
Geometric sequence.
Sn = ____________________
Substitute the values of a1, n and r.
Sn = ____________________
Simplify.

55
Sn = ____________________

4. Independent Practice
A. For each given Geometric sequence, find the sum of the first:
1. 25 terms of 3, 3, 3, ...
2. 50 terms of 4, 4, 4, 4, 4,…
3. 100 terms of -6, 6, -6, 6, -6, 6,….
4. 99 terms of 8, -8, 8, -8, 8, -8, 8,…
5. 6 terms of 32, 64, 128, ...
6. 6 terms of 120, 60, 30, ...
7. 7 terms of 16, 32, 64, ..
8. 7 terms of 27, 9, 3, ...
B. Solve the problem. Show your complete solution.
The game of chess was invented for a Persian king by
one of his servant, Al-Khowarizhmi. Being so pleased, he asked
the servant of what he wanted as a reward. Al-Khowarizhmi
asked to be paid in terms of grain of wheat in a 64 square
chessboard in this manner: 1 grain of wheat in the 1st square, 2
grains in the 2nd, 4 grains in the 3rd, and so on, with the amount
doubling each square until the 64th square. The King was
surprised for the little thing the servant had asked and granted
the servant's request. How many grain of wheat will the servant
be paid?
5. Generalization
To find the sum of the terms in a finite geometric sequence if r ≠ 1, use
Sn = (a1 - anr)/ (1 - r) or
Sn = (a1{1 - rn])/(1 - r)
Where, Sn is the sum of the n-terms
a1 is the first term
r is the common ratio
n is the number of term
To get the sum of a finite geometric sequence if
a. r = 1, use
Sn = a1(n)
b. r = -1 and n is even
Sn = 0
c. r = -1 and n is odd
Sn = a1
6. Application
Solve each problem:
1. Every December, Tagaytay City Science National High School is
sponsoring a Gift-giving program for less fortunate students. A
newspaper fund drive to collect fund was launch. A student
56
promised that he will bring 2 newspapers on the launching day of
the drive, 6 on the second day and triple the number of
newspapers each day until the last day of the fund drive. If the
fund drive is set from December 1 to December 5.
a. How many newspapers will the student bring on the last
day?
b. What is the total number of newspapers that he will
contribute?
2. Rafael is helping his mother in their small “Pasalubong Shop” in
Sky Ranch. If Rafael sold 3 buko tarts in his first day and 6 in his
second day and doubles his sales every day, how many buko
tart did he sell after 10 days?

7. Assessment
Assuming that each of the given geometric sequence is a pyramid
networking. Find the total number of members in each sequence.
1. 5, 15, 45, 135, 405, 1215
2. 2, 8, 32, . . . , 8192
3. 4 , 12, . . ., a9

IV - ASSIGNMENT:
A. Follow up: Sum of Finite Geometric Sequence
1. In the given geometric sequence, the second term is 3 and the sixth
term is 48, find the sum of the first 10 terms.
2. How many ancestors from parents to great-great-great
grandparents do you have?
B. Study: Sum of Infinite Geometric sequence.
1. Differentiate finite geometric sequence from infinite geometric
sequence.
2. What is the formula to find the sum of infinite Geometric
sequence?

57
Answer Key:
Guided Practice;
1. 1,456 4. 37, 324
2. 189 5. 5, 460
3. 126
Independent Practice:
A. 1. 75 5. 2,016
2. 200 6. 945/4
3. 0 7. 2,032
4. 8 8. 1,093/27
B. 1. 18,446,744,073,709,551,615
Application:
1. A) 486 B) 728
2. 3,069
Assessment:
1. 1,820
2. 10,922
3. 39,364
Assignment:
1. Since a2 = 3 and a6 = 48, the value of r = 2. The sum is equal to
3,069.
2. 62

58
Module 1: SEQUENCES
Lesson 2: Geometric and Other Sequences
Lesson 2.2: Infinite Geometric Sequence
Learning Competency 9: Finds the sum of the terms of a given infinite geometric
sequence.

I - OBJECTIVES
a. Determine the ratio of the given infinite geometric sequence.
b. Solve problems involving infinite geometric sequence.
c. Develop analytical mind while doing the assigned task.
II - SUBJECT MATTER
Topic: Sum of Infinite Geometric Sequence
Materials: Activity Sheets, Manila paper
References: TG page 25
LM pages 35-37
Intermediate Algebra by Pastor B. Malaborbor, et.al, pages
317-321
e-math Intermediate Algebra by Orlando A. Oronce, et.al,
pages 444-450
III - PROCEDURE
A. Preliminaries
Motivation:
Another Quick Thinking!
Is it possible to get the sum of …
1. 5, 15, 45, 135, …?
2. 2, 2, 2, 2, 2, 2, …?
3. ¾, 3/8, 3/16 , 3/32 , 3/64 , 3/128 , . . .?
B. Lesson Proper
1. Teaching/Modelling
To get the sum of infinite geometric sequence, the first thing to do
is get the value of r.
If -1 < r < 1 , then it is possible to get the sum.
If r ≥ 1 or r ≤ -1, then it is impossible to get the sum
Illustrative Examples:
a. 5, 15, 45, 135, …
Solution: Since the value of r is 3, then it is impossible to
get the sum.
b. 2, 2, 2, 2, 2, 2, 2,…
Solution: Since the value of r is 1 and there are infinite
terms, it is impossible to get the sum.
c. 3/4, 3/8, 3/16, 3/32, 3/64, 3/128, …
59
Solution: Since the value of r is 1/2, then it is possible to
get the sum. The sum of infinite Geometric sequence can
be obtain by using the formula
Sn = a1 / (1 – r )
Where: a1 = 3/4; r = ½
= (3/4) / ( 1 – ½ )
= 3/2
2. Analysis
1. What is the formula to find the sum of infinite geometric
sequence?
2. How can you determine the value of r in each infinite geometric
sequence?
3. How can you determine if getting the sum of infinite Geometric
sequence is possible or not?
4. Is there another way to find the sum of infinite Geometric
sequence? Explain it briefly.
3. Guided Practice
Find the sum to infinity of each Geometric sequence.
1. 8, 4, 2, 1,…
Solution:
Determine the values of a1, n and r.
a1 = ____; r = _______
Write the formula in finding the sum of infinite
Geometric sequence.
Sn = ____________________
Substitute the values of a1, n and r.
Sn = ____________________
Simplify.
Sn = ____________________

2. 1/2, 1/3, 2/9, 4/27, 8/81, . . .


Solution:
Determine the values of a1, n and r.
a1 = ____; r = _______
Write the formula in finding the sum of infinite
Geometric sequence.
Sn = ____________________
Substitute the values of a1, n and r.
Sn = ____________________
Simplify.
Sn = ____________________

60
3. 16, 8, 4,. . .
Solution:
Determine the values of a1, n and r.
a1 = ____; r = _______
Write the formula in finding the sum of infinite
Geometric sequence.
Sn = ____________________
Substitute the values of a1, n and r.
Sn = ____________________
Simplify.
Sn = ____________________

4. 1/3, 1/9, 1/27, 1/81, …


Solution:
Determine the values of a1, n and r.
a1 = ____; r = _______
Write the formula in finding the sum of infinite
Geometric sequence.
Sn = ____________________
Substitute the values of a1, n and r.
Sn = ____________________
Simplify.
Sn = ____________________

5. 16, 4, 1, ¼, . . .
Solution:
Determine the values of a1, n and r.
a1 = ____; r = _______
Write the formula in finding the sum of infinite
Geometric sequence.
Sn = ____________________
Substitute the values of a1, n and r.
Sn = ____________________
Simplify.
Sn = ____________________

4. Independent Practice
Find the sum of each infinite Geometric Sequence
1. 1/9, 1/27, 1/81, 1/243, . . .
2. 24, 4, 2/3, 1/9, . . .
3. 9, -3, 1, . . .

61
5. Generalization

To get the sum of infinite geometric sequence, get the value of r.


If -1 < r < 1 , then it is possible to get the sum.
If r ≥ 1 or r ≤ -1, then it is impossible to get the sum.
Then use the formula
Sn = a1 /(1 – r) , where –1 < r < 1

6. Application
THINK, PAIR AND SHARE!
Solve each problem.
1. After one swing, pendulum covers 90% of the distance of the
previous swing. If the first swing is 200 centimeters, what is the
total length the pendulum traveled before it comes to a rest.
2. A rubber ball is dropped on a hard surface from a height of 80
feet and bounces up and down. On each rebound, it bounces up
exactly one-half the distance it just came down. How far has the
ball traveled when it appears to come to a stop?
7. Assessment
Solve the problem and show the complete solution.
1. A square is 16 inches on each side. It is positioned to form a new
square by connecting the midpoint of the sides of the
original square. Then two of the corner triangles are shaded. The
process is repeated until the nth time and each time, two of the
corner triangles are shaded. Find the total area of the shaded
region.
2. Christy suffers from allergy once she ate shrimps. Unfortunately
she accidentally ate Palabok with shrimps in Balay na Dako
Restaurant in one of their family bondings. Dr. Diaz of Ospital ng
Tagaytay recommends that she take 300 mg of her medication
the first day, and decrease the dosage by one half each day until
the last day. What is the total amount of medication Christy will
take?

I. ASSIGNMENT
1. Follow-up: Sum of Infinite Geometric sequence
Find the sum to infinity of each Geometric sequence.
1. 5, 5/4, 5/16, 5/64, . . .
2. 2, 1, ½, . . .
3. 6, 2, 2/3, 2/9, . . .
62
2. Study: Harmonic Sequence and Fibonacci Sequence
1. What is Harmonic sequence?
2. What is Fibonacci sequence?
3. Differentiate Harmonic sequence from Arithmetic sequence.

63
Answer Key:
Guided Practice
Find the sum to infinity of each Geometric sequence.
1. 8, 4, 2, 1,…
Solution:
Determine the values of a1, n and r.
a1 = _8___; r = __1/2
Write the formula in finding the sum of infinite
Geometric sequence.
Sn = a1 / (1 – r )
Substitute the values of a1, n and r.
Sn = 8 / (1-1/2)
Simplify.
Sn = 8 /(1/2)
= 16
2. 1/2, 1/3, 2/9, 4/27, 8/81, . . .
Solution:
Determine the values of a1, n and r.
a1 = ½ ; r = __2/3_
Write the formula in finding the sum of infinite
Geometric sequence.
Sn = a1 / (1 – r )
Substitute the values of a1, n and r.
Sn = ½ / (1 – 2/3)
Simplify.
Sn = ½ /1/3
= 3/2
3. 16, 8, 4,. . .
Solution:
Determine the values of a1, n and r.
a1 = __16__; r = ___1/2____
Write the formula in finding the sum of infinite
Geometric sequence.
Sn = a1 / (1 – r )
Substitute the values of a1, n and r.
Sn = 16 /( 1 -1/2 )
Simplify.
Sn = 16 / ½
= 32

64
4. 1/3, 1/9, 1/27, 1/81, …
Solution:
Determine the values of a1, n and r.
a1 = __1/3__; r = ____1/3___
Write the formula in finding the sum of infinite
Geometric sequence.
Sn = a1 / (1 – r )
Substitute the values of a1, n and r.
Sn = 1/3 / ( 1 – 1/3 )
Simplify.
Sn = (1/3) / ( 2/3)
= 1/2
5. 16, 4, 1, ¼, . . .
Solution:
Determine the values of a1, n and r.
a1 = __16__; r = __1/4__
Write the formula in finding the sum of infinite
Geometric sequence.
Sn = a1 / (1 – r )
Substitute the values of a1, n and r.
Sn = 16 / ( 1 – ¼ )
Simplify.
Sn = 16 / (3/4 )
= 64/3
Independent Practice
1. 1/6
2. 144/5
3. 27/4

Application
THINK, PAIR AND SHARE!
1. 2000 centimeters
2. 238.125 feet .
Assessment
1. 16/3 square inches
2. 600 mg
Assignment
Find the sum to infinity of each Geometric sequence.
1. 20/3
2. 4
3. 9

65
Module 1: SEQUENCES
Lesson 2: Geometric and Other Sequences
Lesson 2.3: Harmonic Sequence and Fibonacci
Learning Competency 10: Illustrates the other types of sequences

I - OBJECTIVES:
a. Describe Harmonic and Fibonacci sequences.
b. Solve problems involving Harmonic and Fibonacci sequences.
c. Value the presence of sequence in our daily life.
II - SUBJECT MATTER
Topic: Harmonic Sequence and Fibonacci
Materials: Activity Sheets
References: TG page 26
LM pages 37 – 40
III - PROCEDURE
A. Preliminaries
(Do this as a game.)
GUESS WHAT’S NEXT!
Determine the next term in each sequence
1. S, M, T, ___
2. J, J, A, S, _____
3. J, M, M, J, A, ____
4. 3, 5, 8, 13, ______
5. 2, 6, 18, ______
6. 1, 4, 9, 16, ____
7. 1, 8, 27, ____
8. 4, 7, 12, 19, ____
9. 3, 9, 27, 81, _____
10. 1, ½, 1/3, ¼, 1/5, ____

B. Lesson Proper
1. Teaching/Modelling
Other types of sequences are Harmonic and Fibonacci
Sequences.
Fibonacci sequence is a sequence where its first two terms are
either both 1, or 0 and 1; and each term, thereafter, is obtained by
adding the two preceding terms.
Examples:
1. 1, 1, 2, 3, 5, 8, 13, 21, . . .
2. 0, 1, 1, 2, 3, 5, 8, 13, 21, . . .
Harmonic sequence is a sequence whose reciprocals form an
66
arithmetic sequence.
Examples:
1. 1/24, 1/20, 1/16, 1/12, . . .
2. 4/3, 1, 4/5, 2/3, . . .
In activity # 1, which sequence is Harmonic and which sequence
is Fibonacci?
To solve problems involving Harmonic sequence, convert it into
an Arithmetic sequence by taking the reciprocal of each term. Use
the appropriate formula in the Arithmetic sequence, and then, again
get the reciprocal of the term/s.
There is no formula for the sum of the terms of a harmonic
sequence, simply complete the sequence and add all the terms.
Illustrative example:
1. Find the 12th term of the Harmonic sequence
1/9, 1/12, 1/15, . . .
Solution
Get the reciprocal of each term.
9, 12, 15, . . .
Solve the 12th term of the Arithmetic
sequence using
An = a1 + ( n -1 ) d
Find the values of n, a1 and d.
n = 12 ; a1 = 9 ; d = 3
Substitute the values in the formula and
simplify,
An = 9 + (12 – 1) 3
An = 42
Get the reciprocal.
The 12th term is 1/42
2. Analysis
1. Using Two-Column Chart Method, compare or differentiate
Arithmetic sequence from Harmonic sequence.
2. Is the sum of the Harmonic sequence the reciprocal of the sum of
the arithmetic sequence? Verify your answer.
3. How can you find the nth term of a Harmonic sequence?
3. Guided Practice
Solve each problem.
1. Insert two harmonic means between 6 and 3/2.
Solution:
Get the reciprocals of the terms.
6 → _______
3/2 → ______

67
Given now the Arithmetic sequence, determine the
value of n = ___ ; a1 = ___ ; a4 = ___
Use the formula an = a1 + (n – 1) d, Find the value
of d.
Solve the Arithmetic means using
a2 = a1 +d a3 = a2 + d
Get the reciprocal of each term.
6, ___ , ___ , 3/2

2. Find the sum of 2/3, ½, 2/5, ___,___,___,___, 1/5


Solution:
Get the reciprocals of the terms.
2/3 → _______
1/2 → ______
Given now the Arithmetic sequence, determine the
value of n = ___ ; a1 = ___ ; and d = ____
Complete the Arithmetic sequence by using
a4 = a3 +d a5 = a4 + d
a6 = a5 +d a7 = a6 + d
Write the complete Arithmetic sequence
___, ___, ___, ___, ___, ___, ___, ___
Get the reciprocal of each term.
2/3, ½, 2/5, ___ , ___ , ___ , ___, 3/2
Add all the terms.
Sn = 2/3 + ½ + 2/5 + __ + __ + __+ __ + 3/2

4. Independent Practice
Find the indicated sum in each sequence.
1. 3/2, 6/7, ___ , 6/13
2. 2, 1, 2/3, ___ , ___, 1/3
3. ___ , 1/3, 1/5, 1/7, ___
4. 1, 1, 2, 3, 5, 8, . . . , 55
5. 7, 10, 17, 27, 44, 71, 115. . . , a 10

5. Generalization

Fibonacci sequence is a sequence where its first two terms are


either both 1, or 0 and 1; and each term, thereafter, is obtained by
adding the two preceding terms.
Harmonic sequence is a sequence whose reciprocals form an
arithmetic sequence.
To solve problems involving Harmonic sequence, convert it into
an Arithmetic sequence by taking the reciprocal of each term. Use

68
the appropriate formula in the Arithmetic sequence, and then, again
get the reciprocal of the term/s.
There is no formula for the sum of the terms of a harmonic
sequence, simply complete the sequence and add all the terms.

6. Application
Remember This!
The sequences in column A are all Arithmetic. Supply the
missing terms in Column A and match them in Column B which are
Harmonic. Write the letter that corresponds to the answer in the box.

A B
1. 4, 9, ___, ___, ___ T ___, ___, ___, 4/33, 2/17
2. ___, ___, -1, 1 I ___, 1/3, ___, 1/7, ___
3. 8, ___, ___, -7, -12 R ___, 1, 2/3 , ___
4. ½, ___, ___, ___, 5/2 A ___, -1, -1/9, ___, ___
5. 2, ___, ___, 8, 10 C ___, ___, ___, 1/3, 4/15
6. 1, ___, 5, ___, 9 P ___, ___, ___, 1/19, 1/24
7. ___, 7/6, 5/3, ___, 8/3 S ___, 1/3, -1/2, ___, ___
8. 1, 2, ___, ___, ___, 6 E -1/5, -1/3, ___, ___
9. 7, ___, ___, -17, -25 U 3/2, ___, ___, 6/13, ___
10. 15/2, 31/4, ___, ___, ___ N ___, ¼, 1/6, ___, ___
11. ¾, 3/2, 9/4, ___, ____ V ___, ___, 1/3, ¼, ___, ___

1 9 10 6 2 5 11 2

6 3 9

8 6 4 10 7 2

7. Assessment ( Group of 5 or more.)


Solve by showing your complete solution
One type of rabbit breeds in such a manner that a pair
produces another pair of rabbits at the end of one month. The
next month, the original pair produces another pair and then
stops breeding. All pairs of rabbits of this type breed this way:
give birth to a pair of rabbits on the first and second months and
then stop breeding.(Assuming that none die, and the females
always gives birth to one male and one female.)

69
1. How many pairs of rabbits will be there after
the fourth? Seventh month?
2. How many rabbits are there after one year?

IV - ASSIGNMENT
1. Follow-up: Fibonacci Sequence
Fibonacci Numbers in Nature
( Experimental Procedure)
a. Pick a flower in your garden and count the number of petals. Does the
number of petals equal a Fibonacci number? What is the mane of the
flower?
b. Pick a pineapple and count the number of its “mata”.Is it a Fibonacci
number?
c. Cut a piece of fruit in half so that you create a cross-section. Count the
number of seeds in the fruit. Do you discover any more Fibonacci
numbers?
d. Start your own investigation and list down what part of nature can you
find Fibonacci numbers.

2. Study: Division of Polynomials


a. What are the steps to divide polynomial by another polynomial?

70
Answer Key:
Guided Practice:
Solve each problem.
1. Insert two harmonic means between 6 and 3/2.
Solution:
Get the reciprocals of the terms.
6 → __1/6_____
3/2 → __2/3____
Given now the Arithmetic sequence, determine the
value of n = __4_ ; a1 = _1/6__ ; a4 = _2/3__
Use the formula an = a1 + (n – 1) d, Find the value
of d. d = 1/6
Solve the Arithmetic means using
a2 = a1 +d a3 = a2 + d
= 1/6 + 1/6 = 1/3 + 1/6
= 1/3 = 1/2
Get the reciprocal of each term.
6, __3_ , __2_ , 3/2
2. Find the sum of 2/3, ½, 2/5, ___,___,___,___, 1/5
Solution:
Get the reciprocals of the terms.
2/3 → _ 3/2______
1/2 → _ 2_____
2/5 → ____5/2__
Given now the Arithmetic sequence, determine the
value of n = __8_ ; a1 = __3/2_ ; and d = _1/2___
Complete the Arithmetic sequence by using
a4 = a3 +d a5 = a4 + d
= 5/2 + 1/2 = 3 + 1/2
= 6/2 a5 = 7/2
a4 = 3

a6 = a5 +d a7 = a6 + d
= 7/2 +1/2 = 4 + 1/2
= 8/2 = 9/2
a6 = 4
Write the complete Arithmetic sequence
3/2 , _2_, _5/2, _3_, _7/2_, _4_, _9/2_, _5_
Get the reciprocal of each term.
2/3, ½ , 2/5, 1/3 , 2/7 , 1/4 , 2/9, 3/2
Add all the terms.
Sn = 2/3 + ½ + 2/5 + 1/3 + 2/7 + 1/4+ 2/9 + 3/2
Sn = 5,239/1260

71
Independent Practice:
Find the indicated sum in each sequence.
1. 3,111/910
2. 949/10
3. 563/315
4. 143
5. 1,265

Application:
Remember This!
The sequences in column A are all Arithmetic. Supply the
missing terms in Column A and match them in Column B which are
Harmonic. Write the letter that corresponds to the answer in the box.
A B
1. 4, 9, 14, 19, 24 T ___, ___, ___, 4/33, 2/17
2. -5, -3, -1, 1 I ___, 1/3, ___, 1/7, ___
3. 8, 3, _-2, -7, -12 R ___, 1, 2/3 , ___
4. ½, 1, 3/2, 2 , 5/2 A ___, -1, -1/9, ___, ___
5. 2, 4_, 6_, 8, 10 C ___, ___, ___, 1/3, 4/15
6. 1, 3_, 5, _7, 9 P ___, ___, ___, 1/19, 1/24
7. 2/3 ,7/6, 5/3, 13/6, 8/3 S ___, 1/3, -1/2, ___, ___
8. 1, 2, _3 , _4 , _5 , 6 E -1/5, -1/3, ___, ___
9. 7, - 1_, _- 9 , -17, -25 U 3/2, ___, ___, 6/13, ___
10. 15/2, 31/4, 8, 33/4, 17/2_ N ___, ¼, 1/6, ___, ___
11. ¾, 3/2, 9/4, _3, _15/4 V ___, ___, 1/3, ¼, ___, ___

1 9 10 6 2 5 11 2
P A T I E N C E
6 3 9
I S A
8 6 4 10 7 2
V I R T U E

Assessment:

1. 14 pairs of rabbits after the fourth month. 66 pairs of rabbits after the
seventh month.
2. There are 752 rabbits after one year

72
Module 1: SEQUENCES
Lesson 2: Geometric and Other Sequences
Lesson 2.4: Problem Solving Using Sequences
Learning Competency 11: Solve problems involving sequences

I - OBJECTIVES
a. Formulate and solve real-life problems involving sequences.
b. Create their own problem and solution involving sequences.
c. Develop cooperation while doing the assigned task.
II - SUBJECT MATTER
Topic: Sequences
Materials: Activity Sheets
References: TG page 26
LM pages 43 – 46
Intermediate Algebra by Pastor B. Malaborbor, et.al, pages
317-321
e-math Intermediate Algebra by Orlando A. Oronce, et.al,
pages 444-450
III - PROCEDURE
A. Preliminaries
Motivation
(QUICK THINKING ONLY!)
Suppose the auditorium of the Tagaytay International
Convention Center (TICC) has 20 seats in the first row and that each
row has 2 more seats than the previous row. If there are 30 rows in the
auditorium, how many seats are in the last row?
B. Lesson Proper
1. Teaching/Modeling
To solve real-life problems involving sequences, remember the
words “SEE,PLAN,DO and CHECK”.
Illustrative example #1.
Pacita donates Php50 on the first week to a
charitable institution, Php100on the second week, Php200
on the third week. The amount doubles each week. How
much is her total donation for 10 weeks?
Solution:
SEE - What kind of sequence is involve in the
problem?
50 + 100 + 200 + 400 + . . ., + a10
PLAN – What is the appropriate formula to be
used and the needed values?
73
Sn = [a1(1 - rn)] / (1 – r)
Where a1 = 50: r = 2: n = 10

DO – Perform the indicated operation and


simplify.
Sn = [50 ( 1 – 210)] / ( 1 – 2)
= [50 ( 1 – 1,024) / ( -1)
= [50 ( -1,023) / ( -1)
= 51,150
CHECK – The answer should satisfy all the given
information in the problem
50 + 100 + 200 + 400 + 800 + 1600 + 3200 +
6400 + 12,800 + 25,600 = 51,150

2. Analysis

1. How can you determine if the given problem involve


arithmetic sequence? Geometric sequence?
2. What is the appropriate formula to be used?
3. Is there another way to get the correct answer? Explain
briefly your solution.

3. Guided Practice
Do the following with a partner!
1. To replace the trees destroyed by typhoon Yolanda, the
forestry department of Tagaytay has developed a ten-year plan. The
first year they will plant 100 trees. Each succeeding year, they will plant
50 more trees than they planted the year before.
A. How many trees will they plant during the fifth year?
B. How many trees will they have planted by the end of
the tenth year?
Solution:
Complete the table
No. Of years 1 2 3 4 5 6 7 8 9 10
No. Of trees 100 150 200

What type of sequence is involve in the problem?__________


What formula is appropriate to solve the problem?__________
What are the values that you need to answer the problem?
_________________________
A. Determine the values of a1, n and d.
a1 = ____; n = ______; d = _______
74
Substitute the values of a1, n and d.
An = ____________________
Simplify.
A10 = ____________________
B. Write the formula.
Sn = ____________________
Substitute the values of a1, n and d.
Sn = ____________________
Simplify.
S10 = ____________________

2. A young man gave his wife a gift of Php400 on their


wedding day. To please her, he gave her Php800 on their first wedding
anniversary, Php1,600 on their second wedding anniversary, and so on.
A. How much would she receive on their 9th wedding
anniversary?
B. Compute the total amount the wife had received as
gifts from their wedding day up to their ninth wedding
anniversary?
Solution:
Complete the table
No. Of years 0 1 2 3 4 5 6 7 8 9
Amount of Gift 400 800 1600

What type of sequence is involve in the problem?__________


What formula is appropriate to solve the problem?__________
What are the values that you need to answer the problem?
_________________________
A. Determine the values of a1, n and r.
a1 = ____; n = ______; r = _______
Substitute the values of a1, n and r.
An = ____________________
Simplify.
A9 = ____________________
B. Write the formula.
Sn = ____________________
Substitute the values of a1, n and r.
Sn = ____________________
Simplify.
Sn = ____________________

75
4. Independent Practice
Solve the problem. Show your complete solution.
1. Rico qualified as a basketball varsity player of
Tagaytay City Science National High School. As part of his
training, his coach asked him to run 2km farther each week than
he ran the week before. The first week he ran 3 km. If he keeps
up this pattern, how many km will he be able to run at the end of
the tenth week?
2. Ding wants to make a collage using squares. He cuts
paper squares in such a way that the area of each square is 4
times the area of the preceding square. He makes five squares,
and the area of the first square is 4 sq. Cm. What is the total
area of the squares?
5. Generalization
To solve problems involving sequences:
1. Determine the type of sequence involve in the
problem.
2. Use the appropriate formula.
3. Substitute the needed values and simplify

6. Application
Solve each problem:
1. Your father wants you to help him build a dog house in your
backyard. He says he will pay you Php10 for the first week and add an
additional Php20 each week thereafter. The project will take 5 weeks.
How much money will you earn , in total, if you work for the 5 weeks?
2. View Site Restaurant claims that their product is so good that
people have told their friends about it. Two people heard about the
product the first day. On the second day, these two each told two
friends, and the pattern continued
A. How many people were told about the product on the
sixth day?
B. How many people in all would have heard about the
product at the end of the twelfth day?.
7. Assessment
Solve each problem and show the complete solution.
1. Your science teacher is conducting an experiment about a
certain bacteria. If there are 12 bacteria in the culture at the end of one
day, and that number triples every day, find the total number of bacteria
in the culture at the end of the fifth day?
2. Your mother gives you Php100 to start a “Tipid Impok”” Saving
Account. She tells you that she will add Php20 to your saving account
76
each month, if you will add Php10 each month. Assuming that both of
you will do your part, how much will you save at the end of one year.

IV - ASSIGNMENT
A. Follow up: Problems about Sequences
1. 10 months from now, your parents will celebrate their silver
wedding anniversary and you want to give them a small present. In order to do
that, you start to save Php100 on the first month, Php200 on the second
month, Php300 on the third month, and so on for the period of 11 months.
How much money will you save?
B. Study: Division of Polynomials.
1. What are the steps to divide polynomials?

77
Answers Key:
Guided Practice
Do the following with a partner!
1. Solution:
Complete the table
No. Of years 1 2 3 4 5 6 7 8 9 10
No. Of trees 100 150 200 250 300 350 400 450 500 550

What type of sequence is involve in the problem?


_____arithmetic _____
What formula is appropriate to solve the problem?
A. An = a1 + (n - 1 )d
B. Sn = (n/2) ( a1 + an )
What are the values that you need to answer the problem?
a1, n, an and d
A. Determine the values of a1, n and d.
a1 = 100 n = ___10___; d = ___50____
Substitute the values of a1, n and d.
An = 100 + (10 - 1) 50
Simplify.
A10 = 550
B. Write the formula.
Sn = (n/2) ( a1 + an )
Substitute the values of a1, n and d
Sn = ( 10/2) (100 + 550 )
Simplify.
S10 = 3250
2. Solution:
Complete the table
No. Of 0 1 2 3 4 5 6 7 8 9
years
Amount 400 800 1600 3200 6400 12800 25600 51200 102400 204800
of Gift

What type of sequence is involve in the problem?


Geometric
What formula is appropriate to solve the problem?
A. An = a1 r n-1
B. Sn = ( a1 -anr) / (1 - r)

What are the values that you need to answer the problem?
a1 ; n ; r ; an
A. Determine the values of a1, n and r.
78
a1 = 400 ; n 10 ; r= 2
Substitute the values of a1, n and r.
An = 400( 2 10 - 1 )
Simplify.
A9 = 204,800
B. Write the formula.
Sn =( a1 -anr) / (1 - r)

Substitute the values of a1, n and r.


Sn = [ 400 - 204800(2)] / (1 -2)
Simplify.
Sn = 409,200
Independent Practice
Solve the problem. Show your complete solution.
1. 21 km
2. 484 sq. cm
Application
Solve each problem:
1. Php250
2. a. 64 people
b. 4096 people
Assessment
Solve each problem and show the complete solution.
1. 972 bacteria
2. Php430
3. Your mother gives you Php100 to start a “Tipid Impok”” Saving
Account. She tells you that she will add Php20 to your saving account
each month, if you will add Php10 each month. Assuming that both of
you will do your part, how much will you save at the end of one year.
Php430

79
Module 2: Polynomials and Polynomial Equations
Lesson 1: Division of Polynomial
Lesson 1.1: Long Division
Learning Competency 12: Perform division of polynomials using long division

I - OBJECTIVES
a. State the division algorithm of polynomials.
b. Divide polynomials by another polynomial using long division.
c. Express each quotient using division algorithm accurately and systematically.
II - SUBJECT MATTER
Topic: Division of Polynomials using long division
Materials: Activity Sheets
References: Grade 10 Teacher’s Guide pp. 48 - 50
Grade 10 Learner’s Material pp. 57 – 62
Algebra 2 withTrigonometry by Bettye C. Hall, et. al., pp. 464 - 474
III - PROCEDURE
A. Preliminary
Motivation
(QUICK THINKING ONLY!)
Divide and Write
Example: 19 ÷ 5 = 3 + 4/5 ⟷ 19 = 3( 5) + 4
1. 29 ÷ 5 = _______ ⟷ __________
2. 34 ÷ 7 = _______ ⟷ __________
3. 145 ÷ 11 = _______ ⟷ __________
4. 122 ÷ 7 = _______ ⟷ __________
5. 219 ÷15 = _______ ⟷ __________
B. Lesson Proper
1. Teaching/Modeling
Division Algorithm for Polynomials
If P(x) and D(x) are polynomials and D(x) ≠ 0, then
there exist unique polynomials Q(x) and R(x) such that
P(x) ÷ D(x) = Q(x) + R(x) / D(x)
or P(x) = Q(x) D(x) + R(x)
where, R(x) is either 0 or of degree less than that of D(x).

To divide polynomial by another polynomial using long division


1. Arrange the terms in both the divisor and the dividend in
descending order.
2. Divide the first term of the dividend by the first term of the
divisor to get the first term of the quotient.
3. Multiply the divisor by the first term of the quotient and
subtract the product from the dividend.
80
4. Using the remainder, repeat the process, thus finding the
second term of the quotient.
5. Continue the process until the remainder is zero or the
remainder is of a lower degree than the divisor.
Illustrative example 1.
Divide (x3 + 11x2 – 4x – 9) ÷ (x – 2)
1. Arrange the terms in both the divisor
and the dividend in descending order.

x² + 13x + 22 the first term of the divisor to get the


x - 2 x³ + 11x² - 4x - 9 first term of the quotient.
x³ - 2x² 2. Multiply the divisor by the first term of
13x² -4 x the quotient and subtract the product
13x² - 26x from the dividend.
22x - 9 3. Using the remainder, repeat the
22x - 44 process, thus finding the second term
35 of the quotient.

4. Continue the process until the


remainder is zero or the remainder is
of a lower degree than the divisor.

Therefore:
X3 + 11x2 – 4x – 9 = (X2 + 11x + 22) ( x – 2) + 35
Illustrative example 2.
Divide (x4 + 3x3 – 3x + 2) ÷ (x + 2)

1. Arrange the terms in both the divisor


and the dividend in descending order.

x³ + x² - 2x + 1 2. Divide the first term of the dividend by


x + 2 x⁴ + 3x³ + ox² -3x + 2 the first term of the divisor to get the
x⁴ + 2x³ first term of the quotient.
x³ + 0x 3. Multiply the divisor by the first term of
x³ + 2x² the quotient and subtract the product
-2x²- 3x from the dividend.
-2x² - 4x
x+2 4. Using the remainder, repeat the
x+2 process, thus finding the second term
of the quotient.
0

81
5. Continue the process until the
remainder is zero or the remainder is
of a lower degree than the divisor.

Illustrative example 3.
Divide (30x5 – 50x4 – 21x2 + 32x - 8) ÷ (3x - 5)
1. Arrange the terms in both the divisor
10x⁴ + 0x³ + 0x² - 7x - 1 and the dividend in descending order.
3x - 5 30x⁵-50x⁴-0x³-21x² + 32x - 8
2. Divide the first term of the dividend by
30x⁵-50x⁴
the first term of the divisor to get the
0x⁴+0x³
first term of the quotient.
0x⁴+0x³
3. Multiply the divisor by the first term of
0x³-21x²
the quotient and subtract the product
0x³+0x² from the dividend.
-21x² + 32x
-21x² + 35x 4. Using the remainder, repeat the
-3x-8 process, thus finding the second term
-3x+5 of the quotient.
13 5. Continue the process until the
remainder is zero or the remainder is
of a lower degree than the divisor.

Therefore:
30x5 – 50x4 – 21x2 + 32x - 8 = (10x4 – 7x - 1) (3x - 5) - 15
2. Analysis
1. What are you going to do if some terms of the given
polynomials is/are missing?
2. How can you determine if the answer is correct or not?
3. Is there another way to get the correct answer? Explain
briefly your solution.

3. Guided Practice
Do the following with a partner!
Use long division to find the remainder when the
following polynomials are divided by the corresponding linear
expression
1. (x3 + 7x2 + 15x + 14) ÷ (x + 3)
Solution:

82
1. Arrange the terms in both the divisor
and the dividend in descending order.
2. Divide x3 by x. Multiply (x + 3) by x2.
Subtract and bring down the next
term.
3. Divide 4x2 by x. Multiply (x + 3) by 4x.
Subtract and bring down the next
term.
4. Divide 3x by x. Multiply (x + 3) by 3.
Subtract. Then that’s the remainder.

2. (3x3 - 7x2 + x - 7) ÷ (x - 3)
Solution:
1. Arrange the terms in both the divisor
and the dividend in descending order.
2. Divide 3x3 by x. Multiply (x - 3) by 3x2.
Subtract and bring down the next
term.
3. Divide 2x2 by x. Multiply (x - 3) by 2x.
Subtract and bring down the next
term.
4. Divide 7x by x. Multiply (x - 3) by 7.
Subtract. Then that’s the remainder.

3. (4x5 + 18x4 + 7x2 – x - 100) ÷ (2x + 3)


Solution:
1. Arrange the terms in both the divisor
and the dividend in descending order.
2. Divide 4x5 by 2x. Multiply (2x + 3) by
2x4. Subtract and bring down the next
term.
3. Divide 12x4 by 2x. Multiply (2x + 3) by
6x3. Subtract and bring down the next
term.
4. Divide -18x3 by 2x. Multiply (2x + 3)
by -9x2. Subtract and bring down the
next term.
5. Divide 34x2 by 2x. Multiply (2x + 3) by
17x. Subtract and bring down the
next term.

83
6. Divide -52x by 2x. Multiply (2x + 3) by
-26. Subtract. Then that’s the
remainder.

4. Independent Practice
Divide the given polynomials. Show your complete
solution. And express your answer in the form P(x) = Q(x) D(x) + R(x)
1. (x3 + 2x2 – x - 2) ÷ (x - 1)
2. (x5 + 2x4 + 6x + 4x2 + 9x3 - 2) ÷ (x + 2)
3. (4x3 + 16x2 – 7x - 9) ÷ (2x + 3)

5. Generalization
To divide polynomial by another polynomial using long
division
1. Arrange the terms in both the divisor and the dividend in
descending order.
2. Divide the first term of the dividend by the first term of the
divisor to get the first term of the quotient.
3. Multiply the divisor by the first term of the quotient and
subtract the product from the dividend.
4. Using the remainder, repeat the process, thus finding the
second term of the quotient.
5. Continue the process until the remainder is zero or the
remainder is of a lower degree than the divisor

6. Application
The given polynomial expressions represents the volume
and the height of a Casssava cake sold at Loumar’s Delicacies,
respectively What expression can be used to represent the area
of the base of each Cassava cake?
1. (x3 + 7x2 + 5x – 25) cm3 and (x + 5) cm
2. (2x3 - 13x2 – 5x + 100) cm3 and ( x - 5) cm
3. (6x3 - 23x2 + 33x - 28) cm3 and (3x - 7) cm

7. Assessment
Determine the remainder using long division and show the
complete solution.
1. (x3 + x2 – 22x - 25) ÷ (x + 2)
2. (4x6 + 21x5 - 26x3 + 28x - 10) ÷ (x + 5)
3. (6x3 - 25x2 – 31x + 20) ÷ (3x - 2)

84
IV - ASSIGNMENT
A. Follow up: Dividing Polynomials using long division
1. (4x4 - 2x3 - 15x2 + 9x - 6) ÷ (x - 3)
2. (3x4 + 6x2 + 2x3 + 4x - 4) ÷ (x + 2)

B. Study: Division of Polynomials.


1. What are the steps to divide polynomials using synthetic
division?

85
Answer Key:
Guided Practice:
1. x² + 4x + 3
x³ + 7x² + 15x +
x + 3 14
x³ + 3x²
4x² + 15x
4x² + 12x
3x + 14
3x + 9
5
2.

3x² + 2x + 7
x - 3 3x³ - 7x² + x - 7
3x³ - 9x²
2x² + x
2x² - 6x
7x - 7
7x - 21
14
3.

2x⁴ + 6x³ - 9x² + 17x - 26


2x + 3 4x⁵ + 18x⁴ + 0x³ + 7x² - x - 10
4x⁵ + 6x⁴
12x⁴ + 0x³
12x⁴ + 18x³
- 18x³ + 7x²
- 9x³ - 27x²
34x² - x
34x² + 51x
- 52x - 100
- 52x - 78
-22

86
Independent Practice:
1. X3 + 2x2 – x – 2 = ( x2 + 3x + 2)(x – 1) + 0

2. X5 + 2x4 + 6x + 4x2 + 9x3 – 2 = (X4 + 9x2 – 14x + 34)(x + 2) – 70

3. 4x3 + 16x2 – 7x – 9 = (2x2 + 5x – 11)( 2x + 3) + 24

Application:
1. (x2 + 2x – 5) sq. cm.

2. (2x2 – 3x – 20) sq. cm.

3. (2x2 – 3x + 4) sq. cm.

Assessment:
1. 15
2. - 25
3. - 10

Assignment:
1. 4x4 - 2X3 - 15x2 + 9x – 6 = ( 4x3 + 10x2 + 15x + 54)(x – 3) + 156

2. 3x4 + 4x + 6x2 + 2x3 – 4 = (3X3 - 4x2 + 14x - 24)(x + 2) + 44

87
Module 2: Polynomials and Polynomial Equations
Lesson 1: Division of Polynomial
Lesson 1.2: Synthetic Division
Learning Competency 12: Perform division of polynomials using synthetic division

I - OBJECTIVES
a. Illustrate the process of synthetic division.
b. Divide polynomials P(x) by another polynomial D(x) in the form (x – a) using
synthetic division.
c. Express each quotient using division algorithm accurately and systematically.
II - SUBJECT MATTER
Topic: Division of Polynomials using synthetic division
Materials: Activity Sheets
References: Grade 10 Teacher’s Guide pp. 48 - 50
Grade 10 Learner’s Material pp. 57 – 62
Skillbook in Math IV (BEC) by Modesto G. Villarin, Ed.D., et. al,
pp.80- 81
III - PROCEDURE
A. Preliminary
Motivation
Write each polynomial in descending order of x and give its degree.
1. x3 + x2 – 22x - 25x5 + 2
2. 4x2 + 21x5 - 26x3 + 28x - 10 + 5x4
3. 6 – 31x + 3x3 – 2x4
4. x3 + 7x2 + 5x4 – 25x + 5
5. x3 + 7x2 + 5 – 25x + 5x5

B. Lesson Proper
1. Teaching/Modeling
Division Algorithm for Polynomials
If P(x) and D(x) are polynomials and D(x) ≠ 0, then
there exist unique polynomials Q(x) and R(x) such that
P(x) ÷ D(x) = Q(x) + R(x) / D(x)
or P(x) = Q(x) D(x) + R(x)
where, R(x) is either 0 or of degree less than that of D(x).

To divide polynomial P(x) by another polynomial D(x) in the form


(x – a) using synthetic division
1. Arrange on the line the coefficients of the
polynomial (order is in descending powers). Insert
a zero for the coefficient of the missing power of x.
2. Write a, the divisor, on the left.

88
3. Bring down the first coefficient on the third line.
Multiply the first coefficient by a. Write the product
on the second line below the second coefficient.
4. Find the sum of the product and the second
coefficient then write the sum on the third line
below the product.
5. Multiply this sum by a, add the product to the next
coefficient and write again the new sum on the third
line, and so on.
6. Do the same process until a product has been
added to the final coefficient.
7. The last sum in the third line is the remainder. The
preceding numbers are the numerical coefficient of
the quotient. The quotient is a polynomial of degree
one less than the degree of P(x).
Illustrative example 1.
Divide (6x3 + 11x2 – 4x – 9) ÷ (x + 2)
1. Arrange on the line the coefficients
of the polynomial (order is in
descending powers). Insert a zero
-2 for the coefficient of the missing
6 11 -4 -9 power of x.
-12 2 4 2. Write a, the divisor, on the left.
6 -1 -2 -5
3. Bring down the first coefficient on the
third line. Multiply the first coefficient
by a. Write the product on the
second line below the second
coefficient.

4. Find the sum of the product and the


second coefficient then write the
sum on the third line below the
product.

5. Multiply this sum by a, add the


product to the next coefficient and
write again the new sum on the third
line, and so on.

89
6. Do the same process until a product
has been added to the final
coefficient.

7. The last sum in the third line is the


remainder. The preceding numbers
are the numerical coefficient of the
quotient. The quotient is a
polynomial of degree one less than
the degree of P(x).

Therefore:
6x3 + 11x2 – 4x - 9 = (6x2 – x – 2)(x + 2) - 5
Illustrative example 2.
Divide (x4 + 2x3 – 3x + 6) ÷ (x + 2)
1. Arrange on the line the
coefficients of the polynomial (order
is in descending powers). Insert a
zero for the coefficient of the missing
power of x.
-2 2. Write a, the divisor, on the left.

1 2 0 -3 6 3. Bring down the first coefficient on the


-2 0 0 6 third line. Multiply the first coefficient
1 0 0 -3 12 by a. Write the product on the
second line below the second
coefficient.

4. Find the sum of the product and the


second coefficient then write the
sum on the third line below the
product.

5. Multiply this sum by a, add the


product to the next coefficient and
write again the new sum on the third
line, and so on.

6. Do the same process until a product


has been added to the final
coefficient.

90
7. The last sum in the third line is the
remainder. The preceding numbers
are the numerical coefficient of the
quotient. The quotient is a
polynomial of degree one less than
the degree of P(x).

Therefore:
x4 + 2x3 – 3x + 6 = (x3 – 3)(x + 2) + 12
Illustrative example 3.
Divide (30x5 – 50x4 – 21x2 – 29x - 8) ÷ (3x - 5)
1. Arrange on the line the
coefficients of the polynomial (order
is in descending powers). Insert a
zero for the coefficient of the missing
5/3 power of x.
30 -50 0 -21 -29 -8 2. Write a, the divisor, on the left.
50 0 0 35 10 (Divisor should be in the form x – a)
30 0 0 -21 6 2
3. Bring down the first coefficient on the
third line. Multiply the first coefficient
10 0 0 -7 2 2 by a. Write the product on the
second line below the second
coefficient.

4. Find the sum of the product and the


second coefficient then write the
sum on the third line below the
product.

5. Multiply this sum by a, add the


product to the next coefficient and
write again the new sum on the third
line, and so on.

6. Do the same process until a product


has been added to the final
coefficient.

7. The last sum in the third line is the


remainder.

91
8. Divide the quotient by 3. The
preceding numbers are the
numerical coefficient of the quotient.
The quotient is a polynomial of
degree one less than the degree of
P(x).

Therefore: 30x5 - 50x4 – 21x2 – 29x -8 = (10x4 – 7x + 2)(3x - 5) + 2


2. Analysis
1. What are you going to do if some terms of the given
polynomials is/are missing?
2. Why do you have to change the sign of the constant of the
divisor?
3. Which is easier to perform, long division or synthetic
division? Explain briefly your solution.

3. Guided Practice
Do the following with a partner!
Use synthetic division to find the remainder when
the following polynomials are divided by the corresponding linear
expressions
1. (x3 + 7x2 + 15x + 14) ÷ (x + 3)
Solution:
1. Arrange on the line the coefficients of
the polynomial (order is in
descending powers). Insert a zero for
the coefficient of the missing power of
x.

2. Write a, the divisor, on the left.


(Divisor should be in the form x – a)

3. Bring down the first coefficient.


Multiply the divisor by this number.
Write the product under the second
coefficient. Then add.
4. Repeat the process of multiplying and
adding.
5. Continue until there are no more
coefficients in the dividend. The first
three numbers in the bottom row are
the coefficients of the terms of the

92
quotient polynomial whose degree is
one less than the dividend. The last
number is the remainder.

2. (3x3 - 7x2 + x - 7) ÷ (x - 3)
Solution:
1. Arrange on the line the coefficients of
the polynomial (order is in
descending powers). Insert a zero for
the coefficient of the missing power of
x.

2. Write a, the divisor, on the left.


(Divisor should be in the form x – a)

3. Bring down the first coefficient.


Multiply the divisor by this number.
Write the product under the second
coefficient. Then add.
4. Repeat the process of multiplying and
adding.
5. Continue until there are no more
coefficients in the dividend. The first
three numbers in the bottom row are
the coefficients of the terms of the
quotient polynomial whose degree is
one less than the dividend. The last
number is the remainder.

3. (4x5 + 8x4 + x3 + 7x2 - x - 10) ÷ (2x + 3)


Solution:
1. Arrange on the line the coefficients of
the polynomial (order is in
descending powers). Insert a zero for
the coefficient of the missing power of
x.

2. Write a, the divisor, on the left.


(Divisor should be in the form x – a)

3. Bring down the first coefficient.


Multiply the divisor by this number.

93
Write the product under the second
coefficient. Then add.
4. Repeat the process of multiplying and
adding.
5. Continue until there are no more
coefficients in the dividend. The last
sum in the third line is the remainder.

6. Divide the first five numbers in the


bottom row by 2. The result are the
numerical coefficient of the terms of
the quotient polynomial whose
degree is one less than the dividend.

4. Independent Practice
Divide, using synthetic division. Express your
answer in the form:
Dividend = (Quotient) (Divisor) + Remainder
1. (x3 + 8x2 – 5x - 84) ÷ (x + 5)
2. (2x4 + x3 - 9x2 - x + 6) ÷ (x + 2)
3. (x4 - 5x3 + 11x2 – 9x - 13) ÷ (x - 3)
4. (x4 + 10x3 - 16x - 8) ÷ (x + 2)
5. (3x3 - 15x2 + 7x + 25) ÷ (x - 4)

5. Generalization
To divide polynomial P(x) by another polynomial D(x) in the form
(x – a) using synthetic division
1. Arrange on the line the coefficients of the
polynomial (order is in descending powers). Insert
a zero for the coefficient of the missing power of x.
2. Write a, the divisor, on the left.
3. Bring down the first coefficient on the third line.
Multiply the first coefficient by a. Write the product
on the second line below the second coefficient.
4. Find the sum of the product and the second
coefficient then write the sum on the third line
below the product.
5. Multiply this sum by a, add the product to the next
coefficient and write again the new sum on the third
line, and so on.

94
6. Do the same process until a product has been
added to the final coefficient.
7. The last sum in the third line is the remainder. The
preceding numbers are the numerical coefficient of
the quotient. The quotient is a polynomial of degree
one less than the degree of P(x).

6. Application
GUESS WHO??
Divide using synthetic division. Each problem was given a
corresponding box below. The remainder of these problems are
found in column B. Write the corresponding letter in the box
provided for the question
Column A Column B
1. (2x + 3x2 - 15x – 16) ÷ (x - 3)
3
-4 (B)
2. (x3 + 4x2 – 7x - 14) ÷ (x - 2)
20 (A)
3. (2x3 + 5x2 - 7x - 12) ÷ (x + 3)
4. (x4 - 5x2 - 10x – 12) ÷ (x + 2) 4 (E)
5. (6x3 + 3x2 + 10x + 14) ÷ (2x - 3) 56 (V)
0 (U)

1 2 3 4 5 1

7. Assessment
Tagaytay comes from the phase “taga Itay”. According to
history, what animal did the father and son try to kill in the hill?
Divide using synthetic division, then write the letter
corresponding to the expression in the box containing the
answer.

A x4 – 6x2 + 7x - 6 R x4 – 6x3 + 30x – 9


X+3 X–3

B x3 – 12x2 - 5x + 50 O x3 – 6x2 + 7x + 6
X–2 X–3

X2 – 10x - 25 X2 – 3x - 2 X3 – 3x2 + 3x -2 X3 – 3x2 – 9x + 3


95
IV - ASSIGNMENT
A. Follow up:
1. (4x4 - 2x3 - 15x2 + 9x - 6) ÷ (x - 3)
2. (3x4 + 6x2 + 2x3 + 4x - 4) ÷ (x + 2)

B. Study: Division of Polynomials.


1. What is Remainder Theorem?
2. Without using long division or synthetic division, Is there a way to
get the remainder? Explain your answer.

96
Answer Key:
Guided Practice:
Do the following with a partner!
Use synthetic division to find the remainder when the
following polynomials are divided by the corresponding linear
expressions.
1. (x3 + 7x2 + 15x + 14) ÷ (x + 3)
Solution:
-3
1 7 15 14
-3 -12 -9
1 4 3 5
Therefore:
x2 + 4x + 3 + 5
x+3
2. (3x3 - 7x2 + x - 7) ÷ (x - 3)
Solution:

3
3 -7 1 -7
9 6 21
3 2 7 14

Therfore:

3x2 + 2x + 7 + 14
x-3

3. (4x5 + 8x4 + x3 + 7x2 - x - 10) ÷ (2x + 3)


Solution:

-3/2
4 8 1 7 -1 -10
-6 -3 3 -15 24
4 2 -2 10 -16 14

2 1 -1 5 -8 14
Therefore:

2x4 + x3 – x2 + 5x – 8 + 14
Independent Practice: 2x + 3
97
Divide, using synthetic division. Express your answer in the form:
Dividend = (Quotient) (Divisor) + Remainder
1. (x3 + 8x2 – 5x - 84) = (x2 + 3x -20) (x + 5) + 16

2. (2x4 + x3 - 9x2 - x + 6) =
(2x3 – 3x2 – 3x + 5) (x + 2) – 4

3. (x4 - 5x3 + 11x2 – 9x - 13) =


(x3 – 2x2 + 5x + 6) (x - 3) + 5

4. (x4 + 10x3 - 16x - 8) =


(x3 + 8x2 – 16x + 16) (x + 2) – 40

5. (3x3 - 15x2 + 7x + 25) = (3x2 - 3x – 5) (x - 4) + 5

Application:
GUESS WHO??
Divide using synthetic division. Each problem was given a
corresponding box below. The remainder of these problems are
found in column B. Write the corresponding letter in the box
provided for the question
Column A Column B
1. (2x + 3x2 - 15x – 16) ÷ (x - 3)
3
-4 (B)
2. (x3 + 4x2 – 7x - 14) ÷ (x - 2)
20 (A)
3. (2x3 + 5x2 - 7x - 12) ÷ (x + 3)
4. (x4 - 5x2 - 10x – 12) ÷ (x + 2) 4 (E)
5. (6x3 + 3x2 + 10x + 14) ÷ (2x - 3) 56 (V)
0 (U)

A B U E V A

1 2 3 4 5 1

Assessment:
Tagaytay comes from the phase “taga Itay”. According to
history, what animal did the father and son try to kill in the hill?
Divide using synthetic division, then write the letter
corresponding to the expression in the box containing the
answer.

A x4 – 6x2 + 7x - 6 R x4 – 6x3 + 30x – 9


98
X+3 X–3

B x3 – 12x2 - 5x + 50 O x3 – 6x2 + 7x + 6
X–2 X–3

B O A R
2 3 2 3 2
X – 10x - 25 X2 – 3x - 2 X – 3x + 3x -2 X – 3x – 9x + 3

Assignment:
A. Follow up:
4 3 2 3 2
1. (4x - 2x - 15x + 9x - 6) ÷ (x - 3) = 4x + 10x + 15x + 54 + 168
x- 3

2. (3x4 + 6x2 + 2x3 + 4x - 4) ÷ (x + 2) = 3x3 - 4x2 + 14x - 24 + 44


x+2

99
Module 2 : Polynomials and Polynomial Equations
Lesson 2: The Remainder Theorem and Factor Theorem
Learning Competency 13: Proves the Remainder Theorem and the Factor Theorem

I - OBJECTIVES
a. find the remainder using the Remainder Theorem
b. determine whether (x – r) is a factor of a given polynomial
c. develop patience on how to solve exercises in remainder theorem
II - SUBJECT MATTER
Topic: Polynomials and Polynomial Equations
Sub-topic: Remainder theorem
Materials: Learning Activities and Assessment worksheet
References: Grade 10 Teacher’s Guide pp. 51-54
Grade 10 Learner’s Material pp.70 -81
Workbook in mathematics
III - PROCEDURE
A. Preliminaries

Activity : DECODE MY CODE

Evaluate the polynomial at the given values of x. Next, determine the letter that matches your
answer. When you are done, you will be able to decode the message.

A. P(x) = x3 + x2 + x + 3

x -2 -1 0 1 2

P(x)

message

100
B P(x) = x4 – 4x3 – 7x2 + 22x + 18

x -2 -1 0 1 2

P(x)

message

A. 17 C. –3 E. 5 I. 18
M. 3 N. 78 O. 2 O. 30
P. 6 R. 0 S. –6 T. 23

Guide question:

1. How did you find the value of a polynomial expression P(x) at a given value of x?
2. What message did you obtain?

B. Lesson Proper
1. Teaching Modeling

Activity : Directions: Fill in the blanks with words and symbols that will best complete the
statements given below.
Suppose that the polynomial P(x) is divided by (x – r), as follows:

If P(x) is of degree n, then Q(x) is of degree _____. The remainder R is a constant


because ____________________. Now supply the reasons for each statement in the
following table.
STATEMENT REASON

1. P(x) = (x – r) . Q(x) + R

2 . P(r) = (r – r) . Q(r) + R

3. P(r) = (0) . Q(r) + R

4. P(r) = R

The previous activity shows the proof of the Remainder Theorem:


101
The Remainder Theorem

If the polynomial P(x) is divided by (x – r), the remainder R is a constant and is


equal to P(r).
R = P(r)
Thus, there are two ways to find the remainder when P(x) is divided by (x – r), that is:
(1) use synthetic division, or
(2) calculate P(r).

Similarly, there are two ways to find the value of P(r):


(1) substitute r in the polynomial expression P(x), or
(2) use synthetic division.

2. Analysis

Example 1. Find the remainder when (5x2 – 2x + 1) is divided by (x + 2).

Solution:

a. Using the Remainder Theorem:

P(x) = 5x2 – 2x + 1, r = –2
P(–2) = 5(–2)2 – 2(–2) + 1
P(–2) = 5(4) + 4 + 1
P(–2) = 20 + 4 + 1 = 25

Therefore, the remainder when P(x) = 5x2 – 2x + 1 is divided by x + 2 is 25.

102
b. Using synthetic division:

-2 5 -2 1
-10 24
5 -12 25

Thus, the remainder is 25.


3. Guided Practice

Use the Remainder Theorem to find the remainder when the given polynomial is divided by each
binomial. Verify your answer using synthetic division. Indicate whether or not each binomial is a
factor of the given polynomial.

1. P(x) = x3 – 7x + 5
a. x – 1 b. x + 1 c. x – 2

2. P(x) = 2x3 – 7x + 3
a. x – 1 b. x + 1 c. x – 2

4. Independent Practice

Use the Remainder Theorem to find the remainder R in each of the following.

1. P(x) = 4x4 – 3x3 – x2 + 2x + 1


a. x – 1 b. x + 1 c. x – 2
2. P(x) = 2x4 – 3x3 + 4x2 + 17x + 7
a. 2x – 3 b. 2x + 3 c. 3x – 2
3. P(x) = 8x4 + 12x3 – 10x2 + 3x + 27
a. 2x – 3 b. 2x + 3 c. 3x – 2
5. Generalization

1. What is the relation between the remainder and the value of the polynomial at x = r when the
polynomial P(x) is divided by a binomial of the form x – r?
103
2. How will you find the remainder when a polynomial in x is divided by a binomial of the form
x – r?
3. What happens if the remainder is zero?

6. Application

Use the Remainder Theorem to find the remainder R in each of the following.

1. (x4 – x3 + 2) ÷ (x + 2)
2. (x3 – 2x2 + x + 6) ÷ (x – 3)
3. (x4 – 3x3 + 4x2 – 6x + 4) ÷ (x – 2)
4. (x4 – 16x3 + 18x2 – 128) ÷ (x + 2)
5. (3x2 + 5x3 – 8) ÷ (x – 4)

7. Assessment

Use the Factor Theorem to determine whether or not the first polynomial is a factor of
the second. Then, give the remainder if the second polynomial is divided by the first
polynomial.
1. x – 1; x2 + 2x + 5
2. x – 1; x3 – x – 2
3. x – 4; 2x3 – 9x2 + 9x – 20
4. a – 1; a3 – 2a2 + a – 2
5. y + 3; 2y3 + y2 – 13y +6

IV - ASSIGNMENT

Use remainder theorem to find the missing factor in each of the following. Write
your answers in your notebook.
1. x3 – 8 = (x – 2)(__________)
2. 2x3 + x2 – 23x + 20 = (x + 4)(__________)
3. 3x3 + 2x2 – 37x + 12 = (x – 3)(__________)
4. x3 – 2x2 – x + 2 = (x – 2)(__________)
5. 2x3 – x2 – 2x + 1 = (2x – 1)(__________)

104
Answer Key:
A PRELIMINARIES

x -2 -1 0 1 2

P(x) -3 2 3 6 17

message C O M P A

B P(x) = x4 – 4x3 – 7x2 + 22x + 18

x -2 -1 0 1 2

P(x) -6 -6 18 30 78

message S S I O N

105
MODULE 2 : Polynomials and Polynomial Equations
Lesson 3: Polynomial Equations
Lesson 3.1: Factoring Polynomials
Learning Competency 14: Factors polynomials

I - OBJECTIVES

a. Factor polynomials
b. Use synthetic division in factoring polynomials
c. Appreciate the use of synthetic division in factoring
II. SUBJECT MATTER:

Topic: Polynomials and Polynomial Equations

Sub-topic: Factoring Polynomials


Materials:Learning Activities and Assessment worksheet
References: Grade 10 Teacher’s Guide pp. 54 - 55
Grade 10 Learner’s Material pp. 74 - 76
III - PROCEDURE
A. Preliminary

Activity : THINK , PAIR , SHARE

Use remainder theorem to find the missing factor in each of the following.
1. x3 – 8 = (x – 2)(__________)
2. 2x3 + x2 – 23x + 20 = (x + 4)(__________)
3. 3x3 + 2x2 – 37x + 12 = (x – 3)(__________)
4. x3 – 2x2 – x + 2 = (x – 2)(__________)
5. 2x3 – x2 – 2x + 1 = (2x – 1)(__________)

Guide question
1. What are the other factor of the polynomial equation?
2. How did you arrive at your answer?
3 What processes did you used to get the answer?

B. Lesson Proper

1. Teaching Modeling

Activity : Use synthetic division to show


a. (x + 2) and (3x – 2) are factors of 3x4 – 20x3 + 80x – 48.
b. (x – 7) and (3x + 5) are not factors of 6x4 – 2x3 – 80x2 + 74x – 35
106
2. Analysis
1. Is x +2 a factor of 3x4 – 20x3 + 80x – 48? Why?
2. Is 3x - 2 a factor of 3x4 – 20x3 + 80x – 48? Why?
3. how can we say that x - 7 is a factor of 6x4 – 2x3 – 80x2 + 74x – 35
4 how can we say that 3x +5 is a factor of 6x4 – 2x3 – 80x2 + 74x – 35
.
3. Guided Practice
Determine whether the first polynomial is a factor of the second polynomial

1. x3 – 4x2 + 4x – 3 = (x – 3)(__________)
2. x3 + 2x2 – 11x + 20 = (x + 5)(__________)
3. 3x3 – 17x2 + 22x – 60 = (x – 5)(__________)
4. 4x3 + 20x2 – 47x + 12 = (2x – 3)(__________)
5. 4x4 – 2x3 – 4x2 + 16x – 7 = (2x – 1)(__________)

4. Independent Practice

Used synthetic division to find the factors of the following:

1. x3 – 10x2 + 32x – 32 = 0
2. x3 – 6x2 + 11x – 6 = 0
3. x3 – 2x2 + 4x – 8 = 0
4. 3x3 – 19x2 + 33x – 9 = 0
5. x4 – 5x2 + 4 = 0
5. Generalization

How do we factor polynomials using synthetic division?


Step 1: Arrange the coefficients of P(x) in descending powers of x, placing 0s for the
missing terms. The leading coefficient of P(x) becomes the first entry of the
third row.
Step 2: Place the value of r in the upper left corner. In this example,
Step 3: multiply r with the first coefficient of x the write the product below the 2 nd
coefficient of x then add

107
Step 4 Repeat step 3
Step 5.write the quotient, Note that the exponent of q(x) is one less than the largest
exponent in original equation

6. Application

Used synthetic division to find the other factor the polynomials


1. (2x + x3 + 7x2 – 40) ÷ (x – 2)
2. (6x2 + x3 + 2x +44) ÷ (x + 2)
3. ( x3 + 35 + 9x2 +13x) ÷ (x – 5)
4. (4x3 + 26x +320 +21x2) ÷ (x + 5)
5. (–13x + 2x3 – 5x2 – 15) ÷ (2x + 5)

7. Assessment
1. A rectangular garden in a backyard has an area of (3x2 + 5x – 6) square meters.
Its width is (x + 2) meters.
a. Find the length of the garden.
b. You decided to partition the garden into two or more smaller congruent
gardens. Design a possible model and include mathematical concepts in
your design.
2. If one ream of bond paper costs (3x – 4) pesos, how many reams can you buy for
(6x4 – 17x3 + 24x2 – 34x + 24) pesos?

3. The volume of a rectangular solid is (x3 + 3x2 + 2x – 5) cubic cm, and its height
is (x + 1) cm. What is the area of its base?

IV ASSIGNMENT
Find the factors of polynomial

1. 24 x2 - 22 x - 35
2. 2x3 + 3 x2 - 17 x - 30
3. x3 - 3 x2 - x + 3
4. x3 + 4 x2 - 7 x + 2
5. 18 x3 - 57 x2 - 85 x + 100
6. 6x 2 - 16x + 10 .2(x - 1)(3x – 5
7. 18x 3 +3x 2 - 6x .3x(2x - 1)(3x + 2)

108
Module 2 : Polynomials and Polynomial Equations
Lesson 3: Polynomial Equations
Learning Competency 15: Illustrates polynomial equations

I - OBJECTIVES
a. Determine the roots of polynomial equation
b. Illustrate polynomial equations
c. Appreciate the process of getting the roots of polynomial equation
II - SUBJECT MATTER
Topic: Polynomials and Polynomial Equations
Sub-topic: Polynomial Equations
Materials: activity sheets
References: Grade 10 Teacher’s Guide pp.54 -57
Grade 10 Learner’s Material pp. 82- 86
III – PROCEDURE
A. Preliminaries
Determine the real root(s) of each equation.
1. x – 2 = 0
2. x + 3 = 0
3. x(x – 4) = 0
4. (x + 1)(x – 3) = 0
5. x2 + x – 2 = 0
Guide Question:
1. What do you call the given equations?
2. Describe the roots of an equation.
3. In finding the roots of an equation with degree greater than 1,what have you noticed about
the number of roots? Can you recall a principle that supports this?
4. Describe how to solve for the roots of an equation.
5. How many roots does the equation x2 + 2x + 1 = 0 have?

109
B. Lesson proper
1. Teaching Modeling
Some polynomial equations are given below. Complete the table and answer the questions
that follow. (If a root occurs twice, count it twice; if thrice, count it three times, and so on. The
first one is done for you)

Polynomial Equation degree Real roots of an Number of real


equation roots

1. (x + 1)2(x – 5) = 0 3 -1 (2 times) 5 3
2. x – 8 = 0
3. (x + 2)(x – 2) = 0
4. (x – 3)(x + 1)(x – 1) = 0
5. x(x – 4)(x + 5)(x – 1) = 0
6. (x – 1)(x – 3)3 = 0
7. (x2 – 4x + 13)(x – 5)3 = 0
8. (x + 1)5(x – 1)2 =0
9. (x2 + 4)(x – 3)3 = 0

2 Analysis

1. Is it easy to give the roots of a polynomial equation when the polynomial is expressed as
a product of linear factors? Give a strategy to find roots when a polynomial is expressed as a
product of linear factors.
2. What do you observe about the relationship between the number of roots and the
degree of a polynomial equation? This relationship was discovered by the German
mathematician Karl Friedrich Gauss (1777- 1885).

Fundamental Theorem of Algebra


If P(x) is a polynomial equation of degree n and with real coefficients, then it has at
most n real roots.

110
3. Consider the following polynomial equations. At most how many real roots does each
have?
a. x20 – 1 = 0
b. x3 – 2x2 – 4x + 8 = 0
c. 18 + 9x5 – 11x2 – x23 + x34 = 0

3 Guided Practice

Find the roots of the following polynomial equations by applying the Zero- Product Property.
1. (x + 3)(x – 2)(x + 1)(x – 1) = 0
2. (x + 5)(x – 5)(x + 5)(x – 1) = 0
3. (x + 4)2(x – 3)3 = 0
4. x (x – 3)4(x + 6)2 = 0
5. x2(x – 9) = 0

4. Independent Practice

Determine the real root(s) of each equation.


1. x2(x – 9)(2x + 1) = 0
2. (x + 4)(x2 – x + 3) = 0
3. 2x (x2 – 36) = 0
4. (x + 8)(x – 7)(x2 – 2x + 5) = 0
5. (3x + 1)2(x + 7)(x – 2)4 = 0

5. Generalization

*We can directly solve polynomials of Degree 1 (linear) and 2 (quadratic)


*For Degree 3 and up, graphs can be helpful

111
6.Application
Find all real roots of the following equations. Next, write each polynomial on the left
side of the equation in factored form. Show your complete solutions.
1. x3 – 10x2 + 32x – 32 = 0
2. x3 – 6x2 + 11x – 6 = 0
3. x+ – 2x2 + 4x – 8 = 0
4. 3x3 – 19x2 + 33x – 9 = 0
5. x4 – 5x2 + 4 = 0

7. Assessment

Set up a polynomial equation that models each problem below. Then solve the equation,
and state the answer to each problem.
1. One dimension of a cube is increased by 1 inch to form a rectangular block. Suppose
that the volume of the new block is 150 cubic inches. Find the length of an edge of the
original cube.

2. The dimensions of a rectangular metal box are 3 cm, 5 cm, and 8 cm. If the first two
dimensions are increased by the same number of centimeters, while the third
dimension remains the same, the new volume is 34 cm3 more than the original volume.
What is the new dimension of the enlarged rectangular metal box?

IV. ASSIGNMENT

One of the roots of the polynomial equation is given. Find the other roots.
1. – 2x4 + 13x3 – 21x2 + 2x + 8 = 0; x= -1/2
2. x4 – 3x2 + 2 = 0; x= 1
3. x4 – x3 – 7x2 + 13x – 6 = 0; x= 1
4. x5 – 5x4 – 3x3 + 15x2 – 4x + 20 = 0; x= 2
5. 2x4 – 17x3 + 13x2 + 53x + 21 = 0; x= –1

112
Module 2: Polynomials and Polynomial Equations
Lesson 3: Polynomial Equations
Learning Competency 16: Proves Rational Root Theorem

I - OBJECTIVES
a. describe the roots of equation
b. prove the Rational Roots Theorem
c. develop patience in proving rational roots theorem
II - SUBJECT MATTER
Topic: Polynomials and Polynomial Equations
Sub-topic: Polynomial Equations
Materials: activity sheets
References: Grade 10 Teacher’s Guide pp.54 -55
Grade 10 Learner’s Material pp. 87 - 90
III - PROCEDURE
A. Preliminary
By inspection, determine the number of real roots of each polynomial equation. Roots
of multiplicity n are counted n times.
1. (x – 4)(x + 3)2(x – 1)3 = 0
2. x2 (x3 – 1) = 0
3. x(x + 3)(x – 6)2 = 0
4. 3x (x3 – 1)2 = 0
5. (x3 – 8)(x4 + 1) = 0
B. Lesson Proper

1. Teaching modeling
Complete the table. Verify the given numbers in the last column of the table are rational roots of the
corresponding polynomial equation

113
Polynomial Equation Leading Constant Term
Coefficien Roots
t

1. x3 + 6x2 + 11x – 6 = 0 1 1,2,3

2. x3 – x2 – 10x – 8 = 0 –8 –2, –1, 4

3. x3 + 2x2 – 23x – 60 = 0 1 –4, –3, 5

4. 2x4 – 3x3 – 4x2 + 3x + 2 = 0 2 1/2,–1,1,2

5. 3x4 – 16x3 + 21x2 + 4x – 12 = 0 -12 -2/3,1,2,3

2. Analysis

1. Look at the roots of each polynomial equation in the table. Are these roots in the list of rational
numbers in Question 1?
2. Refer to Equations 1 – 3 in the table. The leading coefficient of each polynomial equation is 1.
What do you observe about the roots of each equation in relation to the corresponding constant
term?
You may have observed that the leading coefficient and constant term of a polynomial equation are
related to the rational roots of the equation. Hence, these can be used to determine the rational
solutions to polynomial equations. This observation is formally stated as the Rational Root Theorem
3. Guided practice
By inspection, determine the number of real roots of each polynomial equation. Roots of
multiplicity n are counted n times.
1. (x – 4)(x + 3)2(x – 1)3 = 0
2. x2 (x3 – 1) = 0
3. x(x + 3)(x – 6)2 = 0
4. 3x (x3 – 1)2 = 0
5. (x3 – 8)(x+ + 1) = 0
4 Independent practice

Find all real roots of the following equations. Next, write each polynomial on the left side
of the equation in factored form. Show your complete solutions.
1. x3 – 10x2 + 32x – 32 = 0
2. x3 – 6x2 + 11x – 6 = 0
3. x3 – 2x2 + 4x – 8 = 0
4. 3x3 – 19x2 + 33x – 9 = 0
5. x4 – 5x2 + 4 = 0

114
4. Generalization
To find the rational roots:

* Use Rational Zeros Theorem to locate possible zeros


* Use calculator to narrow down possible zeros
* Use Synthetic Division to rewrite the function as divisor/quotient
* Use the Zero Product Property to find all real zeros

5. Application

Write TRUE if the statement is true. Otherwise, modify the underlined word(s) to make it
true.
1. The roots of a polynomial equation in x are the values of x that satisfy the equation.
2. Every polynomial equation of degree n has n – 1 real roots.
3. The equation 2x3 – 6x2 + x – 1 = 0 has no rational root.
4. The possible roots of 3x5 – x4 + 6x3 – 2x2 + 8x – 5 = 0 are 3/5 ,3 ,5
5. The only rational root of the equation x3 + 6x2 + 10x + 3 = 0 is 3.
7. Challenge yourself
Find the possible rational roots of
1. y = x5 + 3x2 − 10x – 24.
2. y = 3x7 − 12x3 + 52x2 − 96x + 9.

3. y = x4 − 5x2 + 4.

IV. ASSIGNMENT

Find all the rational zeros of P(x) = x 3 -9x + 9 + 2x 4 -19x 2 .

115
Module 2: Polynomials and Polynomial Equations
Lesson 3: Polynomial Equations
Learning Competency 17: solves polynomial equations

I. OBJECTIVES
a. solve polynomial equations

II. SUBJECT MATTER


Topic: Polynomial Equations
Sub-Topic: Solving Polynomial Equations
Materials: activity sheets
References: G10 Mathematics TG pages 66 - 68 ;
G10 Mathematics LM pages 89 - 93;

III. PROCEDURE
A. Preliminary
By inspection, determine the number of real roots of each polynomial equation.
Note that roots of multiplicity 𝒏 is counted 𝒏 number of times.
1. (𝑥 + 2)(𝑥 − 3)(𝑥 + 1)(𝑥 − 6) = 0
2. 𝑥 (𝑥 − 2)(𝑥 + 3)2 = 0
3. 𝑥 3 (𝑥 3 + 8) = 0
4. (𝑥 3 − 1)(𝑥 2 + 1) = 0
5. 5𝑥 (𝑥 3 − 8)2 = 0

B. Lesson Proper
1. Teaching / Modeling
In solving polynomial equation, we are looking for the value(s) of the variable
that will make the equation true. The solution to an equation is also called the
Roots of the Equation.

Illustrative example:
If a polynomial equation is expressed in factored form, the roots are
easily determined, and it is much easier to solve.
1. Let’s have 𝑥 (𝑥 2 − 4)(𝑥 + 3) = 0
Solution:
𝑥 (𝑥 2 − 4)(𝑥 + 3) = 0
Equate each of the factor to zero, and then solve for 𝑥, that is,
𝑥=0 𝑥2 − 4 = 0 𝑥+3=0
2 𝑥 = −3
𝑥 =4
𝑥 = √4
𝑥 = ±2

Therefore the roots of the polynomial equation 𝑥 (𝑥 2 − 4)(𝑥 + 3) = 0 are


0, 2, −2 and −3

116
The difficulty of finding the roots of polynomial increases when the polynomial
is not expressed in factored form.
2. Let’s solve the equation 6𝑥 4 − 19𝑥 3 − 22𝑥 2 + 7𝑥 + 4 = 0
Solution:
This is a 4th degree polynomial, then it has at most 4 real roots.
The leading coefficient is 6, thus its factors are 1, 2, 3 and 6
The constant term is 4 and its factors are 1, 2, and 4
By the rational root theorem, the possible roots are
1 1 1 2 4
±1, ± , ± , ± , ±2, ± , ± , 𝑎𝑛𝑑 ± 4
2 3 6 3 3

By synthetic division:

Trial 1: 𝑥 = 1
1 6 −19 −22 7 4
6 −13 −35 −28
6 −13 −35 −28 −24

Thus, 𝑥 = 1 is not a root.

Trial 2: 𝑥 = −1
−1 6 −19 −22 7 4
−6 25 −3 4
6 −25 3 4 0

Thus, 𝑥 = −1 is one of the roots, and 6𝑥 3 − 25𝑥 2 + 3𝑥 + 4 = 0 is the


first depressed equation.

Trial 3: 𝑥 = 4
4 6 −25 3 4
24 −4 −4
6 −1 −1 0

Since the remainder is equal to zero, then 𝑥 = 4 is also one of the root,
and 6𝑥 2 − 𝑥 − 1 = 0 is the second depressed equation.

Solving the quadratic equation 6𝑥 2 − 𝑥 − 1 = 0


6𝑥 2 − 𝑥 − 1 = 0
(3𝑥 + 1)(2𝑥 − 1) = 0

3𝑥 + 1 = 0 2𝑥 − 1 = 0
3𝑥 = −1 2𝑥 = 1
1 1
𝑥 = − 𝑥 =
3 2

117
Therefore the roots of 6𝑥 4 − 19𝑥 3 − 22𝑥 2 + 7𝑥 + 4 = 0 are 1, 4,
1 1
− 3 and 2

2. Analysis
1. Is there a relationship between the number of roots and the degree of a
polynomial equation?
2. What are the different Theorems or strategies we can use to solve
polynomial equations?

3. Guided Practice
1. Solve the equation 𝑥 3 + 6𝑥 2 + 11𝑥 + 6 = 0
Solution:
The equation has at most ______ real roots. The leading coefficient is
_____, and its factors are _________ and _________. The constant term
is ______, and its factors are ______, ______, ______, ______, _____,
_____, _____, _____, _____, _____, ______, and ______. The possible
roots of the equation are ____, ____, ____, ____, ____ and
____.

To test if 1 is a root of the given equation, use synthetic division.


1 1 6 11 6

Since the remainder is _________, therefore 1 is _________of the


equation.

Test if –1 is a root of the equation.


-1 1 6 11 6

Since the remainder is _________, therefore -1 is _________of the


equation.

𝑥 3 +6𝑥 2 +11𝑥+6
This implies that = 𝑥 2 + 5𝑥 + 6
𝑥+1
We can obtain the other roots of 𝑥 3 + 6𝑥 2 + 11𝑥 + 6 = 0 by solving for
the roots of 𝑥 2 + 5𝑥 + 6 = 0 by factoring or by using the quadratic
formula.

Its roots are _______ and_________. To check, simply substitute each of


these values to the given equation.

118
Therefore, the real roots of the polynomial equation
𝑥 3 + 6𝑥 2 + 11𝑥 + 6 = 0 are ______, ______and______.

The factored form of the polynomial 𝑥 3 + 6𝑥 2 + 11𝑥 + 6 = 0


is_______________________.

Now, try to solve the equation given below on your own.

4. Independent Practice
Solve the polynomial equation 𝑥 4 − 𝑥 3 − 11𝑥 2 + 9𝑥 + 18 = 0

5. Generalization
A root of a polynomial equation is a value of the variable which makes the
polynomial equal to zero.

In solving polynomial equations, we may use:


a. Zero Product Property
b. synthetic division
c. the Remainder Theorem
d. the Factor Theorem, and
e. the Rational Root Theorem

6. Application
Find the roots of each polynomial equation.
1. 𝑥 3 + 2𝑥 2 − 25𝑥 − 50 = 0
2. 𝑥 4 − 6𝑥 3 − 9𝑥 2 + 14𝑥 = 0
3. 𝑥 4 + 5𝑥 3 + 5𝑥 2 − 5𝑥 − 6 = 0
4. (2𝑥 − 1)(𝑥 + 3)(𝑥 − 2) = 0
5. (𝑥 3 − 8)(𝑥 + 3)2 = 0

7. Assessment
Solve each polynomial equation. Show your complete solution.
1. 𝑥 3 − 2𝑥 2 − 4𝑥 + 8 = 0
2. 𝑥 2 (𝑥 3 − 1)(𝑥 − 4) = 0
3. 𝑥 4 − 𝑥 3 − 7𝑥 2 + 13𝑥 − 6 = 0

IV. ASSIGNMENT
1. Follow Up
Solve the polynomial equation.
a. 𝑥 5 − 7𝑥 3 − 2𝑥 2 + 12𝑥 + 8 = 0

2. Study
LM pages 94 -95, Applying polynomial equations in real-life situations

119
Answer Key:

Guided Practice (Let’s Do This!)


1. Solve the equation 𝑥 3 + 6𝑥 2 + 11𝑥 + 6 = 0
Solution:
The equation has at most ______4 real roots. The leading coefficient is _____,
1 The
4
constant term is ______,
6 and its factors are ______,
1 ______,
2 ______,
3 and
______.
6 The possible roots of the equation are ____,
±1 ____,
±2 ____,
±3 and ____.
±6

To test if 1 is a root of the given equation, use synthetic division.


6
1 1 6 11
18
1 7
24
1 77 18

24 not a root
Since the remainder is _________, therefore 1 is _________of the equation.

Test if –1 is a root of the equation.


6
−1 1 6 11
−6
−1 −5
0
1 5 6
0 a root
Since the remainder is _________, therefore -1 is _________of the equation.
𝑥 3 +6𝑥 2 +11𝑥+6
This implies that = 𝑥 2 + 5𝑥 + 6.
𝑥+1
We can obtain the other roots of 𝑥 3 + 6𝑥 2 + 11𝑥 + 6 = 0 by solving for the roots
of 𝑥 2 + 5𝑥 + 6 = 0 by factoring or by using the quadratic formula.
−3 −2
Its roots are _______ and_________. To check, simply substitute each of these
values to the given equation.

Therefore, the real roots −1 −3


of the polynomial −2
equation 𝑥 3 + 6𝑥 2 + 11𝑥 + 6 = 0 are
______, ______and______.
(𝑥 + 1)(𝑥form
+ 3)( )=0
The factored of𝑥the
+ 2polynomial 𝑥 3 + 6𝑥 2 + 11𝑥 + 6 = 0
is_______________________.

120
Independent Practice (I Can Do This!)
Solve the polynomial equation 𝑥 4 − 𝑥 3 − 11𝑥 2 + 9𝑥 + 18 = 0
Solution:
Possible Roots: ±1, ±2, ±3, ±6, ±9, ±18
Using 𝑥 = −1
−1 1 −1 −11 9 18
−1 2 9 −18
1 −2 −9 18 0

Using 𝑥 = 2
2 1 −2 −9 18
2 0 −18
1 0 −9 0

Using the depressed equation 𝑥 2 − 9 = 0


𝑥2 − 9 = 0
(𝑥 + 3)(𝑥 − 3) = 0
𝑥 = −3 𝑜𝑟 𝑥 = 3
The roots of the polynomial equation 𝑥 4 − 𝑥 3 − 11𝑥 2 + 9𝑥 + 18 = 0 are
−1, 2, −3 and 3

Application (Let’s Do More)


Find the roots of each polynomial equation.
1. −2, −5, 5
2. 0, −2, 7, 1
3. −3, −2, 1
1
4. , −3, 2
2
5. 2, −3, −3

121
Assessment (Challenge Yourself!)
1. 𝑥 3 − 2𝑥 2 − 4𝑥 + 8 = 0

2 1 −2 −4 8

2 0 −8

1 0 −4 0

𝑥2 − 4 = 0 The roots of the polynomial equation


𝑥2 = 4 𝑥 3 − 2𝑥 2 − 4𝑥 + 8 = 0 are 2, −2 and 2.
𝑥 = √4
𝑥 = ±2

2. 𝑥 2 (𝑥 3 − 1)(𝑥 − 4) = 0

𝑥2 = 0 𝑥3 − 1 = 0 𝑥−4=0
𝑥=0 3 𝑥=4
𝑥 =1
𝑥=1
The roots of the polynomial equation 𝑥 2 (𝑥 3 − 1)(𝑥 − 4) = 0
are 0, 1 and 4

3. 𝑥 4 − 𝑥 3 − 7𝑥 2 + 13𝑥 − 6 = 0

2 1 −1 −7 13 −6
2 2 −10 6
1 1 −5 3 0

1 1 1 −5 3

1 2 −3

1 2 −3 0

𝑥 2 + 2𝑥 − 3 = 0 The roots of the polynomial equation


(𝑥 + 3)(𝑥 − 1) = 0 𝑥 4 − 𝑥 3 − 7𝑥 2 + 13𝑥 − 6 = 0
𝑥 = −3 𝑜𝑟 𝑥 = 1 are 2, 1, −3 and 1

122
Module 2: Polynomials and Polynomial Equations
Lesson 3: Polynomial Equations
Learning Competency 18: Solves problems involving polynomials and polynomial
equations

I. OBJECTIVES
a. translate verbal sentences into polynomial equations
b. solve problems involving polynomial equations
c. appreciate the use of polynomials in solving word problems

II. SUBJECT MATTER


Topic: Polynomial Equations
Sub-Topic: Solving Problems Involving Polynomial Equations
Materials: pictures, pen and paper
References: G10 Mathematics TM pages 69-74
G10 Mathematics LM pages 94 – 95
Algebra 2, Prentice Hall pages 496 - 501
III. PROCEDURE
A. Preliminaries
Write a polynomial expression or equation for each of the following using x
as the variable
1. five times a number decreased by four
2. the sum of a number and its square
3. a number decreased by three
4. the difference between six times a number and ten
5. The quotient of nine times a number and seven is equal to eight
more than the number
6. The area of a rectangle with length 2 inches more than the width is
32 square inches.
7. The sum of three consecutive even integers is 60.
8. The volume of a rectangular box with length 3 inches more than the
width, and width 1 inch more than the height is 220 square inches.

B. Lesson Proper
1. Teaching / Modeling
Solving problems can be fun, but wwe don’t know where to
begin, it can be very frustrating. Problem solving skills can be
improved greatly with consistent practice.
Problem solving skills is a process, and consists of several
steps which are applied sequentially.
A. Understand the Problem
Read the problem.
What are the given facts?
B. Plan Your Approach
Choose a strategy
C. Complete the Work
123
Apply the strategy. Use the algebra you know to apply the
strategy to solve the problem
D. Interpret the Results
State your answer then check. Does your answer make
sense? Does it satisfy the conditions of the problem?
Illustrative Example:
In the TLE Class at Trece Martires City National High School, the
boys of G10 – Aguinaldo was asked to build a huge wooden
rectangular container with a volume of 60𝑚3 .The width of the
rectangular container is 2 m less than the length and the height is 1 m
less than the length. Find the dimensions of the container.
Solution:
 Understand the Problem
After reading and understanding the problem, sometimes it is
much easier to understand if we draw a diagram.

height (x-1)

width (x-2)

 Plan Your Approach


Choose a strategy. The strategy to use is to translate the facts in
the problem into an equation. Then solve to find the answer.

Assign variables to represent the unknown


Let 𝑥 represent the length
Then 𝑥 − 2 will be the width and 𝑥 − 1 the height.

𝑙𝑒𝑛𝑔𝑡ℎ 𝑥 𝑤𝑖𝑑𝑡ℎ 𝑥 ℎ𝑒𝑖𝑔ℎ𝑡 = 𝑣𝑜𝑙𝑢𝑚𝑒


𝑥 (𝑥 − 2)(𝑥 − 1) = 60
𝑥 3 − 3𝑥 2 + 2𝑥 = 60
𝑥 3 − 3𝑥 2 + 2𝑥 − 60 = 0

124
 Complete the Work
Solve the equation 𝑥 3 − 3𝑥 2 + 2𝑥 − 60 = 0
Using synthetic division
𝑥=2

2 1 −3 2 −60
2 −2 0
1 −1 0 −60
𝑥 = 2 is not a solution,

Try 𝑥 = 5

5 1 −3 2 −60
5 10 60
1 2 12 0
𝑥 = 5 is one of the solution
Then using the depressed equation and quadratic
formula:
𝑥 2 + 2𝑥 + 12 = 0
−𝑏 ± √𝑏2 − 4𝑎𝑐
𝑥 =
2𝑎
−2 ± √22 − 4(1)(2)
𝑥 =
2(1)

−2 ± √−44
𝑥 =
2
Reject the solutions, since they are not real numbers
Therefore,
𝑥 = 5
𝑥−2 = 3
𝑥−1 = 4

 Interpret the Results


The container is 5 m long, 3 m wide, and 4 m high.
Is the volume of the container 60𝑚3 ?
5 × 3 × 4 = 60

125
60 = 60

2. Analysis
1. How do you solve a problem? Do you follow a step by step procedure?
How do you weigh your options?

2. Can we use polynomial equations in solving word problems?

3. Guided Practice
Find four rational numbers such that the product of the first, third and fourth
numbers is 54. Also the second number is 2 less than the first number, the
third is 5 less than the second, and the fourth is 3 less than the third.

Solution:
 Understand the Problem:
You are asked to find _______ rational numbers that satisfies the given
conditions.

 Plan Your Approach


Let x represent the first number.
Then ______ represent the second number. (The second is 2 less than
the first.)
_____________ represent the third number. (The third is five less than
the second.)
_____________ represent the fourth number. (The fourth is three less
than the third.)

The product of the first, third and fourth number is 54. Therefore, the
equation will be:
___________________________________

 Complete the Work:


Using Synthetic Division, test if 1 is a solution:

To see if there are other


rational solutions, use
the quadratic formula to solve the depressed equation.

126
 Interpret the Results
The numbers are ____, ____, ____, ____.
Is the product of the first, third and fourth numbers 54?

4. Independent Practice
1. One dimension of cube is increased by 1 inch to form a rectangular block.
Suppose the volume of the new block is 150 cubic inches, find the length of
an edge of the original cube?

5. Generalization
Problem solving skills is a process, and consists of several steps which are
applied sequentially.

A. Understand the Problem


Read the problem.
What are the given facts?

B. Plan Your Approach


Choose a strategy

C. Complete the Work


Apply the strategy. Use the algebra you know to apply the strategy to
solve the problem

D. Interpret the Results


State your answer then check. Does your answer make sense? Does it
satisfy the conditions of the problem?

6. Application
Solve completely:
1. Find four consecutive even numbers such that the product of the first, third
and fourth is 2240.
2. In an art class, the students are ask to make and design an open box with
a volume of 64𝑐𝑚3 by cutting a square of the same size from each corner
of a square piece of card board 12 𝑐𝑚 on a side and folding up the edges.
What is the length of a side of the square that is cut from each corner

7. Assessment
Solve completely:
The Yes - O club of TMCNHS launches a recycling campaign. In
support of the program, the G 10 – Newton collected all their waste papers
and constructed two boxes, a cube and a rectangular box. The volume of the
127
cube is 7𝑐𝑚3 more than twice the volume of the rectangular box. The length of
the box is 2cm greater than the length of an edge of the cube, its width is 2 cm
less than the length of n edge of the cube, and its height is 1 cm less than the
length of an edge of the cube. Find the dimensions of the cube and the box.

IV. ASSIGNMENT
1. Follow Up:
Answer Activity 14, Grade 10 Mathematics LM, page 94
2. Study
Polynomial Functions, Grade 10 Mathematics LM, page 99, 106-107

128
Answer Key:

Guided Practice (Let’s Do This!)


Find four rational numbers such that the product of the first, third and fourth
numbers is 54. Also the second number is 2 less than the first number, the third is
5 less than the second, and the fourth is 3 less than the third.

Solution:
 Understand the Problem:
You are asked to find 4 rational numbers that satisfies the given conditions.

 Plan Your Approach


Let 𝑥 represent the first number.
Then 𝑥 − 2 represent the second number. (The second is 2 less than the first.)
𝑥 − 7 represent the third number. (The third is five less than the second.)
𝑥 − 10 represent the fourth number. (The fourth is three less than the third.)
The product of the first, third and fourth number is 54. Therefore, the equation will be:
𝑥 (𝑥 − 7)(𝑥 − 10) = 54
𝑥 − 17𝑥 2 + 70𝑥 − 54 = 0
3

 Complete the Work:


Using Synthetic Division, test if 1 is a solution:
1 1 −17 70 −54

1 −16 54

1 −16 54 0

To see if there are other rational solutions, use the quadratic formula to solve the
depressed equation.
𝑥 2 − 16𝑥 + 54 = 0
−𝑏 ± √𝑏2 − 4𝑎𝑐
𝑥 =
2𝑎
16 ± √(−16)2 − 4(1)(54)
𝑥 = 2(1)

16 ± 2√10
𝑥 = = 8 ± √10
2
Reject, since they are not real numbers
 Interpret the Results
The numbers are 1, −1, −6, −9.
Is the product of the first, third and fourth numbers 54? 𝑦𝑒𝑠

129
Independent Practice (I Can Do This!)
One dimension of a cube is increased by 1 inch to form a rectangular
block. Suppose the volume of the new block is 150 cubic inches, find the
length of an edge of the original cube?
Ans. 5 in.

Application (Let’s Do More)


Solve completely:
1. Find four consecutive even numbers such that the product of the first,
third and fourth is 2240.
Ans. 10, 12, 14, 16
2. In an art class, the students are ask to make and design an open box
with a volume of 64𝑐𝑚3 by cutting a square of the same size from each
corner of a square piece of card board 12 𝑐𝑚 on a side and folding up
the edges. What is the length of a side of the square that is cut from
each corner
Ans. 4cm

Assessment (Challenge Yourself!)


Solve completely:
The Yes - O club of TMCNHS launches a recycling campaign. In
support of the program, the G 10 – Newton collected all their waste papers
and constructed two boxes, a cube and a rectangular box. The volume of
the cube is 7𝑐𝑚3 more than twice the volume of the rectangular box. The
length of the box is 2cm greater than the length of an edge of the cube, its
width is 2 cm less than the length of n edge of the cube, and its height is 1
cm less than the length of an edge of the cube. Find the dimensions of the
cube and the box.
Ans. Cube: s=3 cm
Rectangular box: length = 5 cm, width = 1 cm, height = 2 cm

130

Вам также может понравиться